Sie sind auf Seite 1von 83

Ph.D.

Qualifying Examination
Department of Architecture
University of California, Berkeley

Ioana Chinan
April 14, 2014

Qualifying Exam Committee:


Paul Groth (Exam Chair)
Nezar AlSayyad
Greg Castillo
Ananya Roy
Heba Mostafa

1
1. Field Question: Housing and the Modern State
Historically, housing has always been an essential part of urban development
throughout the world. The modern state and its housing policy have become a central
catalyst for housing provision, shaping the spatial formations of many cities.
Why and when did the state get involved in housing in different parts of the
world? (You dont need to cover the entire globe in your answer! Select four or five
salient examples.)
How have approaches to housing policy and the production of urban space
addressed the problems of social inequity?
In the course of your answer, compare and contrast three major attempts to
operationalize housing policy in the Post World War II era, using examples from the free
market and from planned economies. Where appropriate, use visual illustrations that
help to make your case.
Avoid using Romania in your answer.
Table of Contents
Introduction: Social Equity and the Modern State ....................................................... 2
1 The Welfare State: Embedding Markets .................................................................. 4
1.1 Housing Policy after World War II: The United States and Britain ................................... 8
1.2 The Continental European Welfare State ......................................................................... 11
1.3 The Socialist State and the Central Planning of Housing ................................................. 15
1.4 The Third World, Nation States, and Housing ................................................................. 16

2 The Neoliberal State: Disembedding Markets ....................................................... 18


The Housing Problem: Gentrification and Homelessness ......................................................... 19

3 Post-Neoliberalism: Reembedding Markets ........................................................... 22


Citizenship, Social Movements, and the Just City ..................................................................... 22

Conclusion: In Search of a New Utopia? ...................................................................... 23


Bibliography .................................................................................................................... 27

Introduction: Social Equity and the Modern State


A welfare state can be defined as a country in which the welfare of members of
the community is underwritten by means of state-run social services. These can include
health, education, social insurance, pensions, and housing (which always involves a mix
of state and market activity). Traditionally, the welfare state and its social services are
meant to protect people by fostering a fair distribution of resources and addressing issues
of social inequity. State policy should take account of class stratification, income
inequality, and differential housing opportunities based on race and gender. As I show in
this essay, this has not always been the case.
Historically, there have been different approaches to housing policy in different
parts of the world. For reasons of brevity, in this essay I will concentrate on the liberal
(United States), social democratic (Western Europe), socialist (Soviet Bloc), and mixed
approaches (typically found in post-colonial nation-states of the Third World). I will
focus on the period after World War II, which includes the modernization period of the
1950s and 1960s followed by the shift to neoliberalism in housing policy from the mid1970s through the 1980s. The latest stage, which I refer to as the post-neoliberal phase,
involves in a unique way housing issues that relate to social equity and the idea of the just
city. It deals with different forms of social movements and political action that demand
the active involvement of the state for the purpose of ensuring social equity in housing.
Since my analysis starts with the question why and when the state got involved in
housing policy and how did states grapple with issues of social inequity, it is necessary to
first understand what social inequity is. Therefore as an overarching idea I seek to
understand the relationship between the modern states, housing, and social inequity.

3
Housing policy is central to debates in public administration over the distribution
of resources. The discussion below looks at the concept of social inequity as it is situated
in this context. The liberal ideology claims that private housing is not necessarily a public
good. Dwellings should be private property, which is defined in opposition to the public,
meaning the state. Private housing markets exist, many argue, outside the realm of the
public or the state. I will argue here that historically the state has played a crucial role in
the construction, distribution, and allocation of housing through housing policies that
counteracted social inequity. As I will show in the following sections, even privately
owned housing units must be considered public housing because of the level of state
subsidy (particularly in the US) that homeownership for the middle-class entails.
The concept of social equity, as it will be used in this essay, points to nuances that
complicate our understanding of social equality.1 It refers to challenges to power
relations, attempts to make sense of the individuals intersecting advantages and
disadvantages, cultural and economic cleavages, political disenfranchisement, and
differential access by race and gender to recourses and opportunities.
Policy studies sometimes involve the question, Who gets what? When social
equity becomes an administrative concern, one must add the normative question, Who
ought to get what? It has been suggested that [t]he modern discussion of social equity

As Mary Guy and Sean McCandless explain: To be clear, equity and equality are terms that are often
used interchangeably, and to a large extent, they have similar meanings. The difference is one of nuances:
while equality can be converted into a mathematical measure in which equal parts are identical in size or
number, equity is a more flexible measure allowing for equivalency while not demanding sameness. Mary
E. Guy and Sean A. McCandless, Social Equity: Its Legacy, Its Promise, Public Administration Review
72, no. s. 1 (November 1, 2012): S8, doi:10.1111/j.1540-6210.2012.02635.x.

4
largely began with philosophers reflecting on why societies that had been influenced by
social contract theory still had great inequities.2
The concept of social equity is usefully situated in the context of the hegemony of
global capitalism. My analysis is rooted in the US liberal system of housing policy,
referring also to other forms of housing policy in West Europe, which can be compared
with approaches in the Third World and opposed to the socialist approach of centralized
planning of the Soviet Bloc during the Cold War.

The Welfare State: Embedding Markets

The following discussion will analyze some of the most important historical shifts in
American housing policy. The historical outline is meant to illuminate some of the ways
in which the US government after World War II intervened in housing policy with
specific goals targeting specific outcomes. The European case involved a different
approach, with stronger state intervention in housing provision, and with social concerns
forming the main agenda. The American model of homeownership and the hegemony of
market supported state policy eventually took over in Europe from the much stronger
welfare state.
In Britain as well as in colonial America there were several phases in the
relationship between the colonies and England. In both England and colonial America the
interest of the ruling elite in the welfare of its people started with addressing the needs of
the pauper population. While in England the Poor Law went back to the sixteenth and
2

Ibid.

5
seventeenth centuries, in colonial America, poorhouses dominated the structure of
welfare, or relief, as it was called back then.3 The Poor Law refers to the system of the
provision of social security in the United Kingdom from the sixteenth century until the
establishment of the welfare state in the twentieth. The Poor Law was intended to take
care of those unable to work, to board out children, and to set the poor to work. 4
Equally in colonial America, the social institutions in place were meant to provide
relief for the mentally ill, criminals, and the children of the poor. In colonial America the
poor were institutionalized in outdoor relief or auctioned off to local farmers.5 In both
approaches, the emphasis was on labor, on how to train poor people for work. In both
contexts, welfare institutions of this time were put in place with a rehabilitative vision;
they would suppress intemperance, the primary cause of pauperism, and inculcate the
habit of steady work. 6 But the true concern for social cohesion and public health and the
fear of social unrest on the part of the ruling elite surfaced in the urban slums of the early
industrial city.
The fear of social unrest in the mid-nineteenth century drove the social elite of
Britain to consider what it meant to have to take care of the crowded urban population,
and deal with health dangers and the crime-ridden workers quarters of London. From
Peter Hall7 we learn that the very first pieces of welfare legislation in Britain and the
United States was implemented as a result of the fear on the part of the middle and upper
3

Michael B. Katz, In the Shadow of the Poorhouse: A Social History of Welfare in America (Basic Books,
1996), 3.
4
Gosta Esping-Andersen, The Three Political Economies of the Welfare State, Canadian Review of
Sociology/Revue Canadienne de Sociologie 26, no. 1 (February 1, 1989): 1036, doi:10.1111/j.1755618X.1989.tb00411.x.
5
Katz, In the Shadow of the Poorhouse, 11.
6
Ibid.
7
Peter Geoffrey Hall, Cities of Tomorrow: An Intellectual History of Urban Planning and Design in the
Twentieth Century (Wiley, 2002).

6
classes of social unrest among the poor and working class crowded in the tenements of
London and New York. Specifically, in 1885, the British Royal Commission regulations
and later the 1901 Tenement Act in New York. The recommendations arising from the
Royal Commission were designed to ensure that local authorities use existing powers to
classify, categorize, and determine what was to be done with respect to housing
regulations and provisions. In both cases, moral considerations on the part of men who
believed themselves good middle-class Christians were placed at the heart of the matter.
In both Britain and the United States, the history of state involvement in housing
can be defined in terms of four main phases. The first phase starts with the social
concerns of middle class citizens for the welfare of their city. The charity and social work
phase resulted in the first housing program: tenement reform. This was meant to
address the immorality attributed to the slum dwellers. In New York, the resulting
reforms attempted to fix the conditions of the slums by bringing them up to minimum
housing standards.8 The physical conditions of the tenements were perceived as posing a
threat of disease and contagion, intertwined with moral decay.9 Issues of social equality
and the possibility of achieving it through public housing were not discussed. The
government intervention was minimal and did not address any social problems at any
level.
In England, the 1885 British Royal Commission came up with different and more
substantial solutions by implementing a particular model of public housing. This was

Thomas W. Hanchett, The Other Subsidized Housing: Federal Aid to Suburbanization, 1940s-1960s,
Tenements to the Taylor Homes, 2000, 163.
9
Hall, Cities of Tomorrow; Peter Marcuse et al., Searching for the Just City: Debates in Urban Theory and
Practice (Routledge, Chapman & Hall, 2011).

7
rejected entirely in the American model.10 According to Hall, the Royal Commission of
1885 decided on rehousing workers within the urban frontier of London. However, this
idea was abandoned after the 1890s. 11 While suburbanization in the US was based on
private ownership of a single-family house, the British solution was of a more collective
nature. The council housing was under the authority of the London County Council
(LCC), which administered, built, and distributed housing for the working class. This led
to Englands investing in public transportation to serve the need of communities on the
outskirts of London.12
Ultimately, in both the US and England the liberal ideology and the differential
approach to housing policy based on private property and the market failed to address the
issues of social inequity brought up by the reliance on market mechanisms. As Peter
Marcuse rightly points out, the state actively constructs the market but is not benevolent
or concerned for the poor. It tends either to intervene with indifference, or to be meddling
and incompetent. If the state is involved in both the public and private spheres, then its
intentions must both be made transparent and evaluated.13 The true beginning of housing
policy in United States can only be placed within FDRs New Deal.
The first time in United States history that the middle class was deeply hit by
poverty was during the Great Depression. The New Deal housing policy defined
government welfare efforts up to the advent of World War II. The Housing Act of 1937
allowed for budgetary efforts in public housing, representing the first large-large scale
housing bill in the US. This meant housing that was owned by the government and
10

Hall, Cities of Tomorrow, 39.


Ibid., 50.
12
Ibid., 5152.
13
Peter Marcuse, The Myth of the Benevolent State: Towards a Theory of Housing (Columbia University,
Graduate School of Architecture and Planning, 1978).
11

8
directly subsidized. Together with public housing, slum removal and huge highway
construction were on the way. The first public housing policy determined an uneven
urban geography because municipalities could opt out of building public housing. In
order not to distort the market, slum clearances and massive displacement were not well
addressed. There were no new units added. The municipality provided one unit of
housing for every unit demolished, which did not affect housing supply in any way;
hence it did not disturb the market.
1.1

Housing Policy after World War II: The United States and Britain

Americas success in World War Two boosted the US economy in the wars aftermath,
while Europe was struggling to cope with the wars destruction and crumbling
economies. At this point, the American government embarked upon several decisive
housing bills that defined the way in which urbanization and housing would reshape the
American city. Homeownership was on the rise, which was accompanied by massive
suburbanization and the urban exodus that followed. A major step was taken to help the
returning soldiers by introducing the GI Bill, followed by the Mortgage
Insurance/Appraisal System (HOLC) and Mortgage Financing Reforms, which
introduced new types of mortgages (for 30 years, and fully amortizing, with a mortgage
interest tax deduction). The HOLC established standardized appraisal methods, ranked
neighborhoods according to risk factors, and lumped together things like construction
materials with racial elements and other desirable factors.14 The Federal Housing
Association (FHA) implemented these factors in financing schemes that involved private

14

Hanchett, The Other Subsidized Housing.

9
lenders. These changes all made it more affordable to own than rent. The United States
was on the way of becoming a country of homeowners (Fig.1).

Fig. 1 Homeownership rate in US from 1900 to 2008

15

Loans tied to new construction and additional funding for developers fueled
Greenfield development, whereby suburban residents commuted to cities and became
dependent on their cars. Highway acts and population dispersal led to more driving.16
However, the homeownership ideology was racially exclusionary. People of color
were excluded entirely from suburbs and economic opportunities. FHA delineations,
which served for years as the guiding principles for private lending, made it extremely

15

Alex F. Schwartz, Housing Policy in the United States (Routledge, 2013), 46.
Kenneth T. Jackson, Crabgrass Frontier: The Suburbanization of the United States (Oxford University
Press, 1985).
16

10
difficult for minorities to buy or sell houses.17 Hanchett notes that attempts on the part of
blacks to move to suburbs were met by whites with hostility and at times horrific
violence. Massive government expenditures were framed mainly in terms of economic
reconstruction and market forces.18 Britain after the war took the opposite path. Public
housing became the only option for Londons post-war reconstruction.
The classic Welfare State that developed in England between 1945 and 1976
emerged to preserve stability and to block the socialist threat, and because of the
realization that the market does not guarantee equity. The destruction of the war caused
Londons public administrators to be face with a real revolution: government in
Britain had assumed responsibility for the welfare of the people in a way that would have
been unthinkable in the 1930s. Thus, against the populations desire for single-family
houses, the LCC decided to emphasize high-rise public housing. Hence, between 1945
and 1951 the LCC built 13,072 flats and only 81 houses (). 19 In the following years
the proportion of high-rises in public housing rose from 7% in the late 1950s to 26% in
the mid-1960s. The neoliberal years of 1980s drastically curtailed state investment in
social housing (Fig. 2).20

17

Hanchett, The Other Subsidized Housing'.


Ibid.
19
Hall, Cities of Tomorrow, 240.
20
Ibid., 241.
18

11

Fig. 2 Social Housing trajectory before and after 1980s

1.2

21

The Continental European Welfare State

The smallest cluster of welfare regimes is the social-democratic approach in


housing policy. According to Esping-Anderson, the social-democratic model was the sole
welfare approach that was based on the principles of universalism and the
decommodification of social services.22 The social democrat and the conservative

21

Brian Wheeler, What Future for Social Housing?, BBC News, August 4, 2011,
http://www.bbc.co.uk/news/uk-14380936. Accessed April 13, 2014
22
Esping-Andersen, The Three Political Economies of the Welfare State.

12
model historically didnt support home ownership as Fig. 3.

23

Fig. 3 Here are comparative data from 2004, the last time the OECD updated its numbers.

Jim Kemeny maintains that the welfare system in Sweden is based on the nations
intrinsic social homogeneity based in the middle class, and it developed historically as
such.24 Together with Sweden, other Scandinavian countries like Denmark and Finland
rejected the dualisms of state and market, and working class and middle class, promoting
social equity to a level not otherwise encountered in any of the Western European

23

Jim Kemeny, Divergence in European Welfare and Housing Systems., Housing, Theory & Society 28,
no. 4 (2011).
24
Jim Kemeny, Swedish Rental Housing: Policies and Problems (Birmingham, UK: Centre for Urban and
Regional Studies, University of Birmingham, 1981).

13
countries.25 My discussion here draws from the work of Jim Kemeny in housing tenure in
Sweden.
Kemenys work on housing research is up until today one of the most prestigious
conceptually oriented researcher in international housing theories. His scholarship ranges
from housing studies in Sweden to Australia, focusing on comparative studies on issues
of housing tenure. His strong critique on Anglo-American metanarrative of the
supremacy of owner-occupy housing is well known in the literature. As an acerb
challenger of the empiricist nature of housing research in 1980, Kemeny remained the
sole voice of Marxian and Weberian theory in social science and housing theory.26 His
main aim as a researcher in housing theory was trying to understand why Englishspeaking countries (US, UK and later Australia) were so different form Sweden (where
he was living at the time). Trying to understand housing theories that would explain this
phenomenon, he started to apply interactionism and constructivism to housing. He then
published one of his main books The Myth of Home Ownership27 and then developed his
ideas of dualist and integrated rental systems. Later on he started the discussion of the
association of the welfare state and its role in housing provisions. For Kemeny, both
dualist and integrated rental systems were politically constructed.28
He struggled at that time with questions of how does social interest drive housing
policy. This question remained unanswered up until later in his research endeavor. He

25

Esping-Andersen, The Three Political Economies of the Welfare State, 112.


Chris Allen, Reflections on Housing and Social Theory: An Interview with Jim Kemeny, Housing,
Theory and Society 22, no. 2 (2005): 94107, doi:10.1080/14036090510034608.
27
Jim Kemeny, The Myth of Home-Ownership: Private versus Public Choices in Housing Tenure
(London; Boston: Routledge & Kegan Paul, 1981).
28
Allen, Reflections on Housing and Social Theory.
26

14
later went beyond his interest in macro approaches to housing and started paying more
attention to class hegemony and discourses rather than interaction. Kemeny stressed the
importance of the role of the ideology and political strategy, and he developed the
concept of policy constructivism in order to explain how social structure and housing
policy diverge without paying too much attention to the state. In his later work, Kemeny
argued that: current conceptions of European rental systems in comparative housing
research are implicitly based on profit-driven model of rental markets. [] The
alternative social market model is based on encouraging non-profit rental housing to
compete directly with profit renting in order to dampen rents and provide a source of high
standard housing on secured tenancy terms.29
Kemeny calls this approach to social services in the context of housing the social
market. Following Polanyi, Kemeny calls for the re-embedment of the market within
society. According to Kemeney, social markets represent an extension of Keynesianism,
in which the state tries to balance the effects of the profit-seeking market through
countercyclical measures.
An extreme example of this was found in Communist countries, where profit was
replaced with state control.30 The ideology of liberalism rests upon the belief that
freedom of the individual must be granted above all and that the state must support this
freedom and allow individuals to compete freely on the market. In contrast, both the
Keynesian and Communist models remained under the spell of economic determinism.

29

Jim Kemeny, From Public Housing to the Social Market: Rental Policy Strategies in Comparative
Perspective (London; New York: Routledge, 1995), 5.
30
Jim Kemeny, From Public Housing to the Social Market: Rental Policy Strategies in Comparative
Perspective (London; New York: Routledge, 1995).

15
1.3

The Socialist State and the Central Planning of Housing


During the 1950s and 1960s, forced industrialization in the Communist countries

of Europe increased urban populations, and this in turn required new housing, which the
governments set out to provide. Throughout the Soviet bloc, housing allocation as a topdown approach began at the national level and was delegated to individual counties,
where it was managed by local institutions called construction trusts.31 As Hamiltons
study of Eastern European countries has shown, the socialist neighborhood was intended
to be socially uniform, but because allocation priorities were based not only on need but
also on merit, one of the consequences was residential differentiation. Some
neighborhoods were regarded as more prestigious based on the profession of their
inhabitants: engineers and doctors consistently rank[ed] highest in social esteem.32
The criteria used in housing allocation included: (1) profession (and hence
education); (2) need (or class inversion), which was connected to allocation based on
merit; and (3) political role or status within the Party. Workers were typically placed on a
waiting list. The waiting list was based on a point system: the number of points that ones
family was accorded was regulated according to how long it had already been on the list,
and whether or not it was dispossessed (meaning that their house was about to be
demolished). Priority was also given to families that were living in substandard
conditions or that had more than four children. 33 Tenure was established based on
whether the apartment was state-owned or had been purchased from the state. In the latter
case, the apartment could be sold or bequeathed to family members, but there was a limit
31

John Sillince, Housing Policies in Eastern Europe and the Soviet Union (Routledge, 1990).
Richard A. French and F. E. Hamilton, The Socialist City: Spatial Structure and Urban Policy (Books on
Demand, 1979).
33
Ibid.
32

16
on ownership to one apartment per person or family. Differences also included variations
in size and proximity to the workplace.
In the final year of the socialist era, several of these countries saw a withdrawal of
state resources for housing provision and an increased interest in assembling private and
cooperative funds, which was accompanied by an increased role for the market and a
reduced role for administrators.34
Accordingly, private ownership of state housing stock increased tenfold during
the 1970s and early 1980s. After 1986, however, due to growing speculation and the rise
of inequality in housing, the state sharply curtailed the sales of apartments.

1.4

The Third World, Nation States, and Housing


Post-colonial countries in the Third World developed different forms of welfare

housing programs. In the process of nation-state formation many states adopted a mixed
approach in housing programs, combining state-designed and built public housing and
market-driven private housing.
The socialist model can be found in new nations in North Africa, Egypt, Algeria,
and Libya. All are known for having imported know-how regarding standardized housing
from countries in East Europe.35 AlSayyad argues that after the fall of the colonial world
many of the new nation-states found themselves in need of reconstructing their cultural
identity. These new states were in search of national identity based on shorter-term
34

Sillince, Housing Policies in Eastern Europe and the Soviet Union.


Nezar Al-Sayyad, Culture, Identity, and Urbanism in a Changing World: A Historical Perspective on
Colonialism, Nationalism, and Globalization, in Colonial Modern: Aesthetics of the PastRebellions for
the Future, ed. Tom Avermaete, Serhat Karakayali, and Marion von Osten (Black Dog Publishing Limited,
2009), 7787; D. Vais, Exporting Hard Modernity: Construction Projects from Ceauescus Romania in
the Third World', The Journal of Architecture 17, no. 3 (2012): 43351.
35

17
political interests, and the ideology of struggle emerged as a driving force behind most
nationalist movements. 36 AlSayyad further notes that in their nationalist and
independence phase governments in the developing world were obsessed with modernity.
The construction of public housing was part of this obsession. AlSayyad brings
up the example of Egypt in this regard. Gamal Abdel Nassers nationalist government
built almost the entirety of Egypts public housing stock. But the state public housing
program could not keep up with the urbanization processes this involved. 37 Since public
housing failed to provide for the entire population, squatting and private accommodations
remained for many a welcome addition to public housing. Despite the short-lived
socialist period in Egypt, according to AlSayyad, Nassers policies of centralization and
social provision brought deep change to Egyptian society and closed the socioeconomic
gap between the aristocracy and ordinary Egyptians. 38
Many Third World countries grappled with the construction of national identity
through the built environment. Public housing was an active piece of the project that the
high modernism of the post- World War II era was meant to represent. There were many
cases of public housing provision with nationalist colors. Brasilia, Chandigarh in India,
and Dodoma in Nigeria 39 are all cases of administrative centers established together with
public housing projects. The arrival of globalization to newly created nation-states in the
Third World brought with it the neoliberal ideology. In the next section I will focus on
the neoliberal state, showing how state activities worked in favor of the market, and
with what social consequences.
36

Al-Sayyad, Culture, Identity and Urbanism in a Changing World, 80.


Ibid., 82.
38
Ibid.
39
Lawrence J. Vale, Architecture, Power, and National Identity (Yale University Press, 1992).
37

18

The Neoliberal State: Disembedding Markets

The neoliberal phase historically developed after the financial crises of 1973 has
reached its highest point during the presidency of Ronald Reagan in United States and
Margaret Thatcher in United Kingdom. Both eras are known for the retreat of the state
from regulating market mechanisms and the end of the Keynesian institutional
framework of the welfare state. This study approaches neoliberalism as a political and
economic ideology that is based on the idea of freedom of the entrepreneurial individual
to exchange goods and services on the free market, outside the restriction of
governmental institutions.40 Under the neoliberal ideology housing market should
function separately from the society with a minimum institutional regulations. However,
state subsidies trough tax breaks for the middle class increased steadily since 1977
(Fig.2).

40

David Harvey, A Brief History of Neoliberalism (Oxford University Press, 2005),


http://site.ebrary.com/lib/berkeley/docDetail.action?docID=10180656&p00=spaces%20neoliberalism.

19
Fig. 4 Direct and tax expenditure for housing from 1977 to 2008

41

Neoliberalism promotes privatization, deregulation, and retrenchment of state subsidies


for social services in order to reduce state expenditure. It supports private property and in
the case of housing, indorses homeownership above all other forms of housing tenure. In
polanyian sense, the neoliberal creed is disembbeding the market out of the society,
meaning it creates a dualist system, while commodifying any form of resources from
money to land and labor.42 The exchange value of housing becomes more important than
its use value. In terms of access to housing and spatial and social segregation, this study
briefly touches upon two main housing related outcomes of neoliberal ideology within
the city: gentrification and homelessness.
The Housing Problem: Gentrification and Homelessness
The gentrification process is one of the most common outcomes of market
mechanism and has been a point of contention for many critics of the neoliberal urban
policy. In the context of US, according to Beauregard,43 gentrification has the following
characteristics: renewal of housing markets and revitalization of urban neighborhoods;
restoration of deteriorated urban property, particularly in working class neighborhoods,
usually undertaken by middle or upper class owners; movement of middle class
households into urban areas, causing property values to increase and often having
secondary effects of pricing out poor households. Economic process of valorizing and

41

Schwartz, Housing Policy in the United States, 28.


Karl Polanyi, The Great Transformation (Beacon Press, 1957).
43
Robert Beauregard, The Chaos and Complexity of Gentrification", in Gentrification of the City, ed.
Neil Smith and Peter Williams (Routledge, 2013), 3555.
42

20
devalorizing undervalued urban land markets: landlords, with state assistance, participate
in cycles of investment and disinvestment.
Gentrification is often described as a process of subtle violence: displacement
followed by transformation of the communities. Urban revitalization affects only the
renewed buildings, not people. The real estate industry strives for constant renewal,
gentrification being the poster child of the housing market, justified and encouraged
through numerous discursive constructions. In terms of social equity, class stratification
and gentrification go hand-in-hand.
Certain neighborhoods are ripe for development, transformation, and
improvement. It is thought that older residents are dangerous and should be cleared out,
invoking racialized notions of safety and neighborhood stability. Gentrification is often
seen as part of a process that makes neighborhoods safer, but renders invisible prior
residents. Governments provide assistance to real estate interests, and financial
institutions manipulate land markets.44
Displacement is a change in the class composition of the neighborhood over time.
Once the rents and housing prices are no longer affordable to people with lower incomes,
they cannot move into the neighborhood, and people in the neighborhood will have a
harder time staying. Displacement, whether via urban renewal and the bulldozer or by
market forces, is an act of force.
The lack of incentive for public housing and interdictions on the occupation of
unused building space (e.g., squatting), among other factors, have caused American cities
to be riddled with cases of homelessness. The main causes of homelessness include
44

Robert Beauregard, The Chaos and Complexity of Gentrification, in Gentrification of the City, ed.
Neil Smith and Peter Williams (Routledge, 2013), 40.

21
affordability and the rent versus income imbalance. In addition, the drastic budget cuts of
the neoliberal policies of the Reagan administration in the 1980s contributed to the
closure of many mental institutions. The budget cuts meant the direct release into the
society of persons with severe and persistent mental illnesses.

Fig. 5 The homeless population on a single night in 2005-2008

45

In mid-90s as a result of Clintons welfare reform, the American government


reintroduced welfare term limits, restrictions on benefits for immigrants, and harsher
requirements for people with disabilities. Policies criminalized homelessness and
medicalized it through attributions of mental illness. The new state policy imposed
conditional access to shelter for homeless people based on minimum wage work. Local
administrations showed persistent neglect except in policing areas of the cities occupied
by homelessness.46

45
46

Schwartz, Housing Policy in the United States, 85.


Schwartz, Housing Policy in the United States.

22

Post-Neoliberalism: Reembedding Markets

The analysis presented in this essay follows Gosta Esping-Andersons idea of three
political and economic welfare regimes, to which I have added the socialist welfare state.
Following his idea, I attempt to understand the ways in which welfare states following
different models constructed housing policy and operationalized housing programs. In his
2013 book The Three Worlds of Welfare Capitalism,47 Esping-Anderson attempted to
address the question whether there may be a better welfare regime that lies beyond the
classical approach to social welfare. With the advent of neoliberalism and the retraction
or reengineering of the state in support of market mechanisms, the reconceptualization
of the welfare state is more important than it has ever been. In the world of urban studies,
discussions have surfaced around the newly emergent Asian countries. In general, there
are a series of questions concerning the politics of development, urbanization, and
housing that are rendered urgent by the reliance upon the capitalist path to globalization.
Citizenship, Social Movements, and the Just City
Many of the Global South countries, as well as so-called Asian Tiger economies
(Hong Kong, Taiwan, South Korea, and Singapore), have recently registered economic
growth of unprecedented levels: Major cities in the developing world have become
centers of enormous political investment, economic growth, and cultural vitality, and thus
have become sites for instantiating their countries claims to global significance. These
countries and their main cities aspire to reach the level of the world city such as New
York, London, or Paris. Under the neoliberal creed governments aim to do away with
urban poverty, slums, and poor infrastructure and services. Neoliberal housing policies
47

Gosta Esping-Andersen, The Three Worlds of Welfare Capitalism (John Wiley & Sons, 2013).

23
involve slum clearances and land development carried out for the benefit of the middle
and upper classes to achieve the status of a world-class city. 48 The slum integration and
counter-eviction mechanisms raise the question of power relations between the state and
its citizens.
The new middle and upper classes of the city center are dependent on service
labor by persons from the slum communities. Some neo-Marxist critics of neoliberal
ideology have referred to Henri Lefebvres concept of a Right to the City, which
asserts a collective and equal right for all inhabitants of the city to participate in urban
life, in this privileging use value over exchange value. Following Lefebvre, David
Harvey sees cities as involving a geographic distribution of inequality. For social equity,
he suggests, we should think of modes of redistribution and reciprocity beyond capitalist
accumulation by dispossession.49 Social movements can accordingly be seen as
advocating for the re-embedding of the market through the counter-measures taken by the
state within and under the protective countermovement forces of the society, what
Polanyi calls the double movement.50

Conclusion: In Search of a New Utopia?


The market-driven housing economy has been for a long time facilitated by the
selling of the ideal of homeownership. The suburban American Dream ideology could

48

Aihwa Ong, Introduction: Worlding Cities, or the Art of Being Global, in Worlding Cities, ed. Ananya
Roy and Aihwa Ong (Wiley-Blackwell, 2011), 4,
http://onlinelibrary.wiley.com/doi/10.1002/9781444346800.ch/summary.
49
James L. Greer, Review of Social Justice and the City by David Harvey, Ethics 90, no. 4 (July 1,
1980): 6047.
50
Polanyi, The Great Transformation.

24
only be realized by those who could pay enough for it for its suppliers to make a profit.
As seen above, starting in the 1980s, in the United States and most of the West European
countries, housing policies made homeownership possible mainly through private
speculative markets. At the same time, governments withdrew themselves not only from
the public housing sector but also from regulating the market. In the 1990s, the mortgage
market shifted from being dominated by savings and loans to being led by mortgage
companies and investment banks.
In this new neoliberal system regulation was declared unnecessary and little was
done to adapt its supervision mechanisms to the new market structure. As in the case of
the American housing crisis , the fact that subprime mortgage access had been bestowed
upon the least advantaged individuals rendered them helpless under the effects of
financial speculation and an unprotected housing market. Since anyone could get a
mortgage, home purchases and housing prices skyrocketed. The result was the financial
bubble that triggered the global financial crisis of 2008 (Fig. 4).51
Now that we can begin to look beyond the neoliberal creed, what is there to be
done? Re-embedding markets necessitates a stronger welfare state that supports and
guarantees social equity, identifies limitations of resources, and allows for sustainable
development. Housing provision and housing policy should and must account for
exclusionary market practices, and treat access to affordable housing as a right.

51

Schwartz, Housing Policy in the United States.

25

Fig. 6 Household debt-to-income ratio of selected countries (above) and the real house prices leading to
52
the Great Recession (below) in 2002 to 2010

Harvey sees all individuals involved in the production of space as insurgent


architects. The insurgent architect should change the world: As crafty architects bent
on insurgency we have to think strategically and tactically about what to change and
52

Vishaan Chakrabarti, A Country of Cities: A Manifesto for an Urban America (Distributed Art Pub
Incorporated, 2013).

26
where, about how to change what and with what tools. However, we also have to live in
this world. This is a fundamental dilemma that faces everybody interested in progressive
change.53 Accordingly, it is important to find tools of social justice that can help bring
about the end of social inequity within the city. A crafty architect of a space should
consider urban space as a quintessential locus for asserting citizenship rights. In turn, the
insurgent architect must function within the qualitatively different but related areas of
social and ecologic life. 54 Harvey suggests the need to return to and reconsider this
approach. The discussion on the just city, social equity, and the state leads us, according
to Harvey, to the question: Are we in need of a new utopia?

53
54

David Harvey, Spaces of Hope (University of California Press, 2000), 233.


Ibid.

27

Bibliography
Allen, Chris. Reflections on Housing and Social Theory: An Interview with Jim
Kemeny. Housing, Theory and Society 22, no. 2 (2005): 94107.
doi:10.1080/14036090510034608.
Alsayyad, Nezar. Cairo: Histories of a City. Harvard University Press, 2011.
AlSayyad, Nezar. Culture, Identity and Urbanism in a Changing World: A Historical
Perspective on Colonialism, Nationalism and Globalization. In Colonial
Modern: Aesthetics of the Past - Rebelions for the Future, edited by Tom
Avermaete, Serhat Karakayali, and Marion von Osten, 7787. Black Dog
Publishing Limited, 2009.
Beauregard, Robert. The Chaos and Complexity of Gentrification". In Gentrification of
the City, edited by Neil Smith and Peter Williams, 3555. Routledge, 2013.
Chakrabarti, Vishaan. A Country of Cities: A Manifesto for an Urban America.
Distributed Art Pub Incorporated, 2013.
Esping-Andersen, Gosta. The Three Political Economies of the Welfare State.
Canadian Review of Sociology/Revue Canadienne de Sociologie 26, no. 1
(February 1, 1989): 1036. doi:10.1111/j.1755-618X.1989.tb00411.x.
. The Three Worlds of Welfare Capitalism. John Wiley & Sons, 2013.
French, Richard A., and F. E. Hamilton. The Socialist City: Spatial Structure and Urban
Policy. Books on Demand, 1979.
Greer, James L. Review of Social Justice and the City by David Harvey. Ethics 90, no.
4 (July 1, 1980): 6047.
Guy, Mary E., and Sean A. McCandless. Social Equity: Its Legacy, Its Promise. Public
Administration Review 72, no. s1 (November 1, 2012): S5S13.
doi:10.1111/j.1540-6210.2012.02635.x.
Hall, Peter Geoffrey. Cities of Tomorrow: An Intellectual History of Urban Planning and
Design in the Twentieth Century. Wiley, 2002.
Hanchett, Thomas W. The Other Subsidized Housing: Federal Aid to Suburbanization,
1940s-1960s. Tenements to the Taylor Homes, 2000, 16377.
Harvey, David. A Brief History of Neoliberalism. Oxford University Press, 2005.
http://site.ebrary.com/lib/berkeley/docDetail.action?docID=10180656&p00=spac
es%20neoliberalism.
. Spaces of Hope. University of California Press, 2000.

28
Jackson, Kenneth T. Crabgrass Frontier: The Suburbanization of the United States.
Oxford University Press, 1985.
Katz, Michael B. In the Shadow of the Poorhouse: A Social History of Welfare in
America. Basic Books, 1996.
Kemeny, Jim. Divergence in European Welfare and Housing Systems. Housing,
Theory & Society 28, no. 4 (2011).
. From Public Housing to the Social Market: Rental Policy Strategies in
Comparative Perspective. London; New York: Routledge, 1995.
. From Public Housing to the Social Market: Rental Policy Strategies in
Comparative Perspective. London; New York: Routledge, 1995.
. Swedish Rental Housing: Policies and Problems. Birmingham, Eng.: Centre for
Urban and Regional Studies, University of Birmingham], 1981.
. The Myth of Home-Ownership: Private versus Public Choices in Housing
Tenure. London; Boston: Routledge & Kegan Paul, 1981.
Marcuse, Peter. The Myth of the Benevolent State: Towards a Theory of Housing.
Columbia University, Graduate School of Architecture and Planning, 1978.
Marcuse, Peter, James Connolly, Johannes Novy, Ingrid Olivo, and Cuz Potter.
Searching for the Just City: Debates in Urban Theory and Practice. Routledge
Chapman & Hall, 2011.
Ong, Aihwa. Introduction: Worlding Cities, or the Art of Being Global. In Worlding
Cities, edited by Ananya Roy and Aihwa Ong, 126. Wiley-Blackwell, 2011.
http://onlinelibrary.wiley.com/doi/10.1002/9781444346800.ch/summary.
Polanyi, Karl. The Great Transformation. Beacon Press, 1957.
Schwartz, Alex F. Housing Policy in the United States. Routledge, 2013.
Sillince, John. Housing Policies in Eastern Europe and the Soviet Union. Routledge,
1990.
Vais, D. Exporting Hard Modernity: Construction Projects from Ceauescus Romania
in the Third World. The Journal of Architecture 17, no. 3 (2012): 43351.
Vale, Lawrence J. Architecture, Power, and National Identity. Yale University Press,
1992.
Wheeler, Brian. What Future for Social Housing? BBC News, August 4, 2011.
http://www.bbc.co.uk/news/uk-14380936.

1
2. Methods Question: Housing in the First, Second, and Third Worlds

Identify and discuss three significant methodological approaches for studying housing
practices, policies, and the resulting architecture. Your answer should examine the
strengths and limitations of your choice of methodological approaches, by employing
case studies from different countries that represent different stages of development. (If
possible, do not re-use examples that you used in Question 1.)
Explanations for such housing practices have often been associated with
modernization in the First, Second, and Third Worlds. Specifically, what role has the
rise of scientific expertise played in architecture and urban planning in these three
parts of the world?
As with the prior question, use illustrations to help support your answer, and avoid using
Romania in your answer.
Table of Contents
Introduction: Modernity, Housing, and the Expert ...................................................... 2
1. Historiography ........................................................................................................... 3
1.1. A History of a Building Typology: The Panelk .......................................................... 6
1.2. A History of the Profession: The Communist Architect............................................. 9
Strengths and Limitations ...................................................................................................... 11

2. Ethnography ............................................................................................................. 11
2.1. Experts and l'Homme Moyen ....................................................................................... 12
2.1. The Modern City and Its Residents ............................................................................. 13
Strengths and Limitations ...................................................................................................... 16

3. Case Study ................................................................................................................ 17


3.1. Single Case Study: Life on the Street .......................................................................... 18
3.2. Multiple Case Studies: Urban Centralities .................................................................. 20
Strengths and Limitations ...................................................................................................... 23
Conclusion ................................................................................................................................. 24
Bibliography ............................................................................................................................. 26

Introduction: Modernity, Housing, and the Expert


Marshall Berman has said that modernity is a mode of vital experience
experience of space and time, of the self and others, of lifes possibilities and perilsthat
is shared by men and women all over the world today. 1 In other words, various groups
of people experienced a change, at a particular moment in time and confined within a
particular space (a city, a region, or a country). According to Berman, this experience of a
rupture or sudden break with the past is a defining characteristic of modern existence.
Such rupture challenges individuals, and calls into question their identities and the ways
they position themselves against the other (e.g., as modern versus traditional).
Moreover, the change can bring at the same time a positive and a negative result,
in what Berman calls the dialectic of modernization and modernism. 2 Therefore he
further states, to be modern is to find ourselves in an environment that promises us
adventure, power, joy, growth, transformation of ourselves and the worldand, at the
same time, that threatens to destroy everything we have, everything we know, everything
we are. 3
I start this essay with Marshall Bermans poetic description of modernity because
it underlines several of the points that I wish in this essay to bring together. The
modernist movement in architecture and urban planning represents a quintessential aspect
of a vast body of scholarship. In the interwar period and the post World War Two era, the

Marshall Berman, All That Is Solid Melts Into Air: The Experience of Modernity (Verso, 1983), 15.
Ibid.
3
Ibid., 16.
2

3
political and economic project of modernization, in housing policy, urban planning, and
architectural practice, fundamentally redesigned many cities throughout the world.
The modernist movement in architecture and its political and social agenda attempted to
reshape society and the individuals within it. Housing projects played a crucial role in
this humanistic and technocratic project. Planners and architects were the forerunners of
this process. Some of the authors mentioned in this study are critical of technocrats
involvements, while others are apologetic. Ultimately, what the essay attempts to unravel
are the methods used by the authors under discussion, in order to make sense of this
complex and intricate process called modernization, including its social and spatial
constructions, and its technocrats and their purpose: the transformation of the human
being.

1. Historiography
In her book History in Practice Ludmilla Jordanova starts by identifying several
meanings of the word history. She states: The word history has a number of
meanings, and a wide range of connotations, some of which are charged with intense
emotion. We use it to invoke the authority of precedents, to refer to what is no longer
relevant, to endow objects with value and status, and to mobilize longings for better
worlds. Since one of the main meanings of history is simply the past, then almost any
association with the past times can be transferred to history. 4 Indeed, addressing the
past, identifying historical facts that are representative, defining periods and ultimately
constructions of the historians subjective understanding of the past, represent the
4

Ludmilla Jordanova, History in Practice (Bloomsbury USA Academic, 2013), 1.

4
repertoire of the discipline of history. How then can we think of the author, his choices of
historical facts, and modes of emplotment in writing architectural history? Why do
historians choose distinct buildings as historical facts or advance a particular architect as
quintessential and representative of a style or a movement?
My study here will address historiography in the context of modern architecture. I
will attempt to construct the how of a particular form of modernity, that of the post
World War Two period in Eastern Europe, chiefly with reference to modern architecture
in former Czechoslovakia. I will bring as an example the work of Kimberly Elman
Zarecor and her book Manufacturing a Socialist Modernity: Housing in Czechoslovakia,
1945-1960. 5 I have chosen this book as an historiographic example for three reasons:
(1) architectural historical fact: the standardized structural panel buildings (panelk) as
used for collective housing in Communist Czechoslovakia; (2) the concern for
architecture practice within the Communist system and the consequences of this practice
for housing design and policy in the socialist era; and (3) Zarecors position as an
historian outside the vilifying rhetoric about socialist standardized mass housing in
Eastern Europe.
In one of her earlier articles, Zarecor argues that the socialist block of flats in the
post-war era was not an expression of raw technicality, or of a preoccupation with
quantity than quality and aesthetic sensibility, but a symptom of local cultural, political
and economic changes in Czechoslovakia which recalibrated the relationship between
artistic creativity and technological determinism in design, altering the role of architects

Kimberly Elman Zarecor, Manufacturing a Socialist Modernity: Housing in Czechoslovakia, 1945-1960


(University of Pittsburgh Press, 2011), 218.

5
in society, their relationship to the structures of power, and the types of buildings they
produced. 6 Her book followed the same line of argument, highlighting the continuity
between the pre-Communist and Communist eras, the place of the architect in serving the
public, and accepting ones role as an architect who had been stripped of his individuality
and creativity in order become a mere technocrat in the massive project of housing
provision in the postwar period. The author specifically points to the local specificity of a
new type of Czechoslovak modern architecture subjected to historical contingencies and
defined independently of the Soviet leadership of that time.
Zarecor focuses on the period from the start of the Kosice program in 1945 to the
end of the Second Five-Year Plan in 1960. She explains: These events are points of
entry and exit along a continuum of architectural modernism in Czechoslovakia.7
Zarecors method is a compilation of different approaches to historical facts. She uses
archival materials and the only architectural journal in her country at the time,
Czechoslovak Architecture. Primary sources are from various institutions (like the
Stavoprojekt research institutes and the Institute of Prefabricated Buildings), appeals to
the authority of secondary sources, and an enormous collection of photographic
material8 of plans and buildings. She pays particular attention to the architects Vladimr
Karfk and Jir Kroha and their role in the design and production of mass housing.
Housing policy and architecture go hand-in-hand. The fast pace of industrialization not
only demanded housing new workers in increasingly expanding industrial cities, but also
6

Kimberly Elman Zarecor, The Local History of an International Type: The Structural Panel Building in
Czechoslovakia, Home Cultures 7, no. 2 (July 1, 2010): 21735,
doi:10.2752/175174210X12663437526250.
7
Zarecor, Manufacturing a Socialist Modernity, 6.
8
Ibid., 4.

6
addressed the severe housing shortage of the post-war period. Zarecor argues that
architects working for the socialist housing project after 1948 were already invested in
functionalism, along with various political and social commitments. For Zarecor, their
pledges were only a continuation of something they had already accepted: a social agenda
that architects took upon themselves in the pre-socialist era. Zarecor constructs her
overall historical narrative along two lines. One is a history of a building typology in post
World War II era and the other is the transformation of architectural practice centered on
several preeminent figures in modern architecture of the time.
1.1. A History of a Building Typology: The Panelk
To follow the narrative of building typology, Zarecor looks first at the short-lived
socialist realism of the early post-war era. She starts with the example of the OstrovaPoruba housing units, the so-called T-series housing. These housing units were built in
the early 1950s and they were still under the spell of socialist realism (Fig. 1 left). The
Stavoprojekt Institute together with the Ministry of Technology and Education
constructed the first housing project for miners in the city of Ostrava. The Model
Housing Developments in Ostrava after 1948 was planned as a large complex with
apartment houses, schools, and a town square offering a department store, house of
culture, health clinic, post office, butcher, and groceries store.9 Unfortunately, because
of budget overruns the Czechoslovak Building Works seized the project in early 1950.

Ibid., 95.

Fig. 1 Archway on the main street in Poruba,


Ostrova; apartment building showing socialist
realist style (left); Buildings containing
T1 type units in the Prosek neighborhood
of Prague (right)

10

The next housing model Zarecor presents is the T-series (Fig1, right). This new
housing model represents a more standardized version, which was based on several
typologies under study by Stavoprojekt and the Typification Institute along with the
Department of Housing.11 Following the Ostrova model, Zarecor takes the narrative to a
more advanced era of typification. The next phase in housing policy and design was the
industrialization of housing, the evolution of panelk: particularly experimentation
with new industrial building technologies and housing prototypes.12 The housing
program was instituted by the new Ministry of Building Industry with the coordination of
the Stavoprojekt institute. The new model of panel technology modular housing was
meant to achieve a faster design and construction process and be more cost-effective.
10

Ibid., 104, 168.


Ibid., 97.
12
Ibid., 224.
11

8
Prefabricated housing became the topic of extensive research by architects and engineers
in the early 1950s. The typification efforts of this period brought the BA system13 (Fig.2):
The technology was notable for its use of pre-stressed concrete frames that were filled
with lightweight, unreinforced concrete to create a single panel.14

Fig. 2 BA system prototype Bratislava 1955

13

15

The BA system was named for Bratislava since the system was first used there. Zarecor further explains
that the Bratislava project (see figure 2) won the best housing design in 1955. Ibid., 274.
14
Ibid., 275.
15
Ibid.

The next generation of panelk was the G-series, which allowed for a faster and more
sophisticated construction process, while still bearing the marks of socialist realism: relief
columns at corners, detailed cornices, and pitched roofs. The later versions of the same
G-series were further stripped of the stylish details reminiscent of socialist realism, and
revealed a return to the functionalist creed of early modernism. Housing units in this
phase were at the core of all the criticism that followed in the late socialist and postsocialist periods.
1.2. A History of the Profession: The Communist Architect
In the second line of her narrative Zarecor follows the transformation of the
architecture profession by looking at several figures whom she deems representative for
this period and at the architectural profession overall. Zarecor constructs her story by
presenting the development of the profession as collectively unified and mediated, but at
the same time acknowledging the importance of several outstanding figures who were
representative of ways in which architects were embedded within the political system. At
the same time Zarecor does not charge the architect with the political stain of the
Communist creed. She craftily depoliticizes the technocrat, architect, and engineer,
arguing that each was perceived as part of a collective force within the Stavoprojekt
Institute.
The author specifically follows the career path of Jir Kroha. Zarecor identifies Kroha as
an important cultural figure who provides a rare opportunity for an in-depth study of the

10
implementation of socialist realism in Czechoslovakia and the region.16 Kroha was
important not only for his cultural contributions, but also for his ties within the
Communist Party and his support for the Soviet architecture of the time. To support this
the author uses personal correspondence between Kroha and Communist Party President
Zdenek Nejedly.17 Accordingly, Zarecor follows Krohas career by showing plans,
elevations, and images that portray the architects contribution to the built environment
between 1945 and 1950.
The second individual whom Zarecor identifies as an important cultural figure,
specifically important for the technological advancement of the standardized housing
units, is Vladimr Karfk. He was responsible for the innovation of a hybrid system with
pre-stressed concrete frames embedded in the panels. This system is associated with the
G-series of mass housing. This first prototype was part of the temporary exhibition at the
1952 Architects Congress in Prague.18 The exhibition was the outcome of the research
program condudcted by the Institute of Building Materials and Construction in
Bratislava. Zarecor looks at Karfiks memoir An Architect Remembers, to understand the
importance of the BA system. Here she also shows the connection between the Soviet
experimental housing research conducted by architect A. Michailov in early 50s. Karfik
continues his research in prefabricated panels after his meeting with Michailov in
Moskow. The brief attention that Zarecor gives to Karfik versus Kroha is telling. While
Kroha is presented throughout the length of one chapter, Karfik takes one page and he is
just the one technocrat, among others involved in the prefabricated panel systems: Karel
16

Ibid., 178.
Ibid.
18
Ibid., 274.
17

11
Janu, Adolf Bens briefly mentioned. This approach is telling of the ways in which
Zarecor looks at the research of prefabricated technology for housing as a collective
effort.
Strengths and Limitations
Historical research taps into sources from the past, combining primary and
secondary sources, emplotted by the historian in a narrative that suggests interpretations
of the past, which might be relevant to imagining a possible future. The mode of inquiry
of historical research defines a style of documentation that should be grounded in the
hard facts of the archival past, which still exist in the present as unquestionable
artifacts.
The limitation of historical research lies within its interpretative character. The
subjectivity and positionality of the historian filters facts through the way in which order,
importance, and personal agenda are established. Sometimes the story itself can only be
evaluated by situating it within multiple stories that support a particular historical event
or fact.

2. Ethnography
In discussions of ethnography, one controversial aspect has been the issue of
representation and positionality. On the one hand, some have argued that ethnographys
biggest weakness concerns problems of representation, because of the holistic approach

12
to scientific inquiry.19 On the other hand, some argue that participatory observation
should account for the subjectivity of the inquirer and the different voices that comprise
an ethnographic representation.20 The dialogical experiment in ethnography, as Marcus
calls it, falls short precisely because of its uncritical engagement with representation. He
states, The key recurrent problem in pursuing this strategy is () a sense of corruption
involved in the description of the oral in the production of ethnography.21
2.1. Experts and l'Homme Moyen
In French Modern22 Paul Rabinow argues that modern discourses situates society
in a constant flux and that temporary stability is accomplished through a rationality that
involves standardization, mechanization, and efficiency, and that this intervention must
happen by transforming individuals. But in order to do so first one must identify the
homme moyen, the universal subject, who is able to be transformed in a new subject, and
the ways in which he can be transformed. The rationalization of space is the means to
achieve l'homme moyen. New forms and norms must be envisioned. The prvoyance
(prediction) of the future must be based on reforming the individual, the state, space, and
society. The ultimate target of reform is the society.23 Norms and forms in Rabinows
sense are the realm of the new experts: administrative officials, city planners, architects,
and engineers. The modern city was the realm of the expert.

19

George E. Marcus, Ethnography Through Thick and Thin (Princeton University Press, 1998); James
Clifford and George E. Marcus, Writing Culture: The Poetics and Politics of Ethnography (University of
California Press, 1986).
20
Clifford Geertz, Thick Description: Toward an Interpretive Theory of Culture, The Interpretation of
Cultures, 1973, 330; Clifford and Marcus, Writing Culture.
21
Marcus, Ethnography Through Thick and Thin, 37.
22
Paul Rabinow, French Modern: Norms and Forms of the Social Environment (University of Chicago
Press, 1995).
23
Ibid., 169.

13
2.1. The Modern City and Its Residents
Ethnographic work, frequently used in sociology, anthropology, and cultural
studies, plays an important role in qualitative research in architecture studies. It involves
a mode of emplotment that concentrates on the study down inquiry, looking at
practices, interrelationships, and the cultural constructions of individuals in their natural
settings. It focuses primarily on empirical material and is also frequently cited in the
research methods literature as [involving] an underlying emphasis on an inductive
process. 24
As an example of the use of ethnographic research in the context of the
relationship of housing to modernity, I will use the case of Brasilia, analyzed by James
Holston in his book The Modernist City: An Anthropological Critique of Brasilia.25 Here
Holston employs different tactics, including photo-documentation, architectural
inventories, place-centered observations, interviews with existing residents, and statistical
data (official census data) gathered from state institutions.
Holstons book example is representative for the ways in which he engages with
people, specifically Brazilians living in the newly designed housing units of the utopic
city of Brasilia. The study describes what the author argues was a failed experiment in
social engineering. He looks not only at Brasilias inhabitants but also to its technocrats:
engineers, office workers, and architects. The most interesting part of Holstons study is
his skillful shift from study up to study down. He not only unravels the lack of social
life in the streets and the problems of collective modern living, but also inquires into the
24

Linda N. Groat and David Wang, Architectural Research Methods (John Wiley & Sons, 2013), 218.
James Holston, The Modernist City: An Anthropological Critique of Brasilia (University of Chicago
Press, 1989).
25

14
mechanism of nation-state crafting: The merging of state and society that the modernist
model presupposes: the identification of the state as the organizer of social life, through
work, in every sector of society. Thus, even though the functions are spatially separated
into homogenous zones, they are each motivated, organized, and regulated by the same
planning agent, the state.26
The ethnographic work that Holston presents is the result of his engagement with
the everyday life of the new pioneers, the public servants for whom the city was
intended to begin with, and the construction workers excluded from its modern spaces.
Holstons anthropological study engages with modernism critically. With reference to a
concept of everydayness, it critiques the built environment by pointing to the
shortcomings of utopia: failure in homogenizing the society, the marginalization of
workers housing through the construction of a workers periphery outside the
administrative center, and the denial of access to the city for lower-level technocrats and
construction laborers.
The city of Brasilia was planned based on an idea of modernist functional rationality,
residential uniformity, and monumentality. These are hallmarks of modernism. But there
is a social dimension to this. Each building brings together inhabitants of various classes,
in a building whose faade hides their status differences and invites social integration.
The result, however, differed from the premise of the project. The vision of its creators
proved untenable in a historically deeply segregated society. To show this, Holston
presents the attempt of Niemeyer to design some of the apartments with differential
means of access: different elevators and entrances for service workers versus the owner.
26

Ibid., 154.

15
There was also the attempt to integrate traditional spaces as copa,27 (Fig. 3) which
eventually became a space mainly associated with the kitchen but also equally accessed
by every occupant.

Fig. 3 Hostons two examples of copas and kitchens in the middle and upper-middle income apartments.
The two plans are showing the separation between servant access area and the main private and public
areas of the apartment. The servant and the main family side of the apartment were separated and accesses
shouldnt intersect.

27

According to Holston, Copa began to appear on house plans at the turn of the century to identify the
large cupboard in the passageway between the kitchen and the veranda that was used to store utensils,
cooking spices, tea, biscuits, ripening fruit, vials of medicine, needle and thread in short the infinite trivia
of household life. Ibid., 177.

16
The shift in scale from the design of the housing unit to the administrative plaza
and its monuments is telling. The public monuments are at the core of the city: the
ministries, embassies, and hotels. The bureaucratic centrality was telling of ways in
which allocation of housing and land was done according to the master plan.

Fig. 4 The superquadra housing blocks on the North side (left) and the South (right). These images show
the scale of the buildings in relationship with its landscape surroundings and the open space.

Strengths and Limitations


Holstons account of his engagement with the inhabitants of superquatras I found
to be unclear with respect to who his informants really were. He only mentions his
conversing with a class of nine-year-olds, asking them to draw a house. The houses
they drew resembled single family, pitched roof dwellings, not the typical flat roof box
of the modernist utopia.28 This example is followed by a long list of statements from
Holstons interviews showing the discontent of the residents living in the superquadras.
There are no voices of contentment. The possible counterpoint seems to be either left out
or inexistent. The nature of Holstons informants is undisclosed: no class, no gender, and

28

Ibid., 171.

17
no political color, just a mass of unhappy residents stripped of their traditional living
environments.29

3. Case Study
According to Robert Yin, a case study is comprised of two parts: first, [a] case
study is an empirical inquiry that investigates a contemporary phenomenon within its real
life context, especially when the boundaries between phenomenon and context are not
clearly evident.30 Secondly, [a] case study inquiry copes with the technically distinctive
situation in which there will be many more variables of interest than data points, and as
one result relies on multiple sources of evidence, with data needing to converge in a
triangulation fashion, and as another result benefits from the prior development of
theoretical propositions to guide data collection and analysis.31 To put it in other words,
a case study comprises the logic of design, the means and techniques of collecting data,
and data analysis. A case study can be single, as in the case of Jane Jacobss book The
Death and Life of Great American Cities concentrating on the case of New York City, or
multiple, as in the case of Laurence Vales Architecture, Power, and National Identity. In
this section I will concentrate on Vales book, looking at his cases of capital cities: Abuja
in Nigeria, and Dodoma in Tanzania.

29

Ibid., 172.
Robert K. Yin, Case Study Research: Design and Methods (Sage, 2009), 18.
31
Ibid.
30

18
3.1. Single Case Study: Life on the Street
A case study can be single, as in the case of Jane Jacobss book The Death and
Life of Great American Cities,32 concentrating on New York City, or multiple, as in
Laurence Vales Architecture, Power, and National Identity. In this part of the analysis, I
will discuss the difference between Vales multiple case study approach and Jane
Jacobss study of New Yorks urban spaces. In discussing Vales book, I will concentrate
on two cities designed in the 1960s: the capital cities of Abuja, the new capital of Nigeria,
and Dodoma in Tanzania.
Jane Jacobss critique of modernism is well-known in architecture and planning
theory. The book attacks modern urban planning, advocating a return to the traditional
American city. This is the city of the street, of small neighborhoods, single-family
houses, and local commercial spaces. Jacobss writings were an important influence on
the New Urbanism architecture and planning movement that emerged in the 1980s.
Jacobs envisions four generators of traditionalism: active streets with mixed functions,
short blocks allowing for pedestrian free movement, a variety of buildings in age and
level of repair, and ultimately lower density. The author advances this agenda through an
acerbic critique of the modernization projects carried out by Robert Moses in New York
during the Great Depression. Jacobs provides many examples of the socio-physical
dynamics of everyday life in New York. As she puts it in the introduction: The way to
get at what goes on in the seemingly mysterious and perverse behavior of cities is, I
think, to look closely, and with as little expectations as possible, at the most ordinary

32

Jane Jacobs, The Death and Life of Great American Cities (Random House Digital, Inc., 1961).

19
scenes and events, and to attempt to see what they mean and whether any threads of
principles emerge among them.33
In order to build her critique Jacobs defamiliarizes the familiar, her own New
York neighborhood. Jacobs looks at problems found in other cities. She mentions the
fictional mixtures in Pittsburgh, speculation about street safety in Philadelphia and
Baltimore, meanderings of downtown in Boston, and the unmaking of slums in
Chicago.34 She continues: In every case, I have tried to test out what I saw or heard in
one city or neighborhood against others, to find how relevant each citys or each places
lessons might be outside its own special case.35
Jacobs wants to rediscover the street and save it from the modernist emptiness;
she wants to bring back a different modernism. In Bermans words, much of the
meaning for which modern men and women were desperately searching, in fact, lay
surprisingly close to home, close to the surface and immediacy of their lives: it was all
right there, if we could only learn to dig.36 Jacobss idea of the urban montage37
romanticizes the urban space, and attempts to rediscover diversity, contradictions, and
urban vitality.
Jacobss single case study is rich in details, nuances, observation techniques,
interviews, and schematic diagrams. She does not use any photographs, for which she
explains her reasoning behind this thus: The scenes that illustrate this book are all about

33

Ibid., 13.
Ibid., 15.
35
Ibid., 16.
36
Berman, All That Is Solid Melts Into Air, 315.
37
Ibid., 314.
34

20
us. For illustrations, please look closely at real cities. While you are looking, you might
as well also listen, linger and think about what you see.38 All the illustrations that we
might need already exist within ourselves, and if not we should all create them with our
gaze by looking outside ourselves into the streets of our own cities.
3.2. Multiple Case Studies: Urban Centralities
Unlike, Jacobs single case study, Architecture Power and National Identity is
made out of a multitude of case studies. Some cases are standing on their own and others
presented in a comparative perspective. In his book, Vale argues that capitol buildings in
post-colonial contexts are the product of competing political and cultural forces, and that
more often those buildings are carriers of meanings in the service of the newly
established nation-state, but that in the end regimes built capitol complexes chiefly to
serve some personal, subnational, and supranational interests rather than to advance
national identity; designers cannot mold political change; and governments still find it
necessary to demonstrate their power through aesthetic exaggerations.39
In this book Vale presents case studies of different capitols in developing
countries, tracing back in time capitol cities in Europe, and identifying some of the main
criteria of political and symbolic power. Vale points to the traditional from of cities
like Paris, London, and Washington; to capitals that were renewed like Athens, Rome,
and Moscow; and in the post-World War II era, to high modernist capitals like
Chandigarh and Brasilia, and more recently, Islamabad, Dodoma, and Abuja.

38
39

Jacobs, The Death and Life of Great American Cities.


Lawrence J. Vale, Architecture, Power, and National Identity (Yale University Press, 1992), 293.

21
Compared with the other case studies, these latter two capital cities contained designed
housing units, and they each took different approaches to residential solutions and their
relationships with the political power.
Both Abuja and Dodoma are expressions of post-colonial independence. Both are
products of rapid modernization, with a clear social agenda: achieving social unity. For
Abuja the master plan proposed a centralized city, with an institutional core and two side
extensions of housing units, similar with Brasilias bird shape (Fig. 5). The residential
units, however, are only intended for middle and upper class part of the population.

Fig. 5 Compared with Dodoma, Abuja shows an ordered capital city along a main axis with emphasis on
40
government institutions.

40

Ibid., 141.

22

41

Fig. 6 Dodomas functional arrangements. It attempts to be a non-hierarchical capital city.

Vale concentrates on the political and cultural meaning of the institutions


designed at the core of the city without too much attention given to housing. At the same
time, the modernization creed for social equality falls short in Abuja. Any form of
housing policy completely excluded the poor from the city. To support his claims of
spatial structuring of the city, in both cases, Vale shows several plans and sketches of
functional hierarchy, photos of the model and renderings showing the public spaces
situated on the main axis. Unlike Abuja, in the case of Dodoma the residential area is
more substantial (Fig. 6). Located around the administrative core, the residential was one

41

Ibid., 150.

23
of the priorities in planning decision-making. Monumentality was not a priority for the
master plan. Government emphasized human habitation.42

Strengths and Limitations


Many of the critiques of case study method deal with the problem of
generalization to theory. How can one case study be used to create a theory? Yin strongly
contests this limitation and asserts that applying the theory to other cases can validate
findings within a particular case study. The problem of causation, therefore, is
fundamental to case studies: Jacobs sidewalks, parks, the mix uses, and the need for
small blocks generates a particular human conduct within the city. Moreover, as this case
is well known for the ways in which it has revolutionized planning theory and
represented a great contribution to the field.43
The multiple case study deals to the issue of how many cases should be enough to
generate a theory? In Vales discussion of capital cities the relationship between
architecture, power and national identity is manifold: firstly he looks at historical
developments of major capitals of the world in Europe and US, then he extrapolates to
Third World countries identifying the political and social agenda of the post-colonial
world. The last four post-colonial capitol complexes deal with the way in which political
powers in search for national identity produce architecture, more precisely how political
power generates and negotiates a plurality of meanings.

42
43

Ibid., 151.
Yin, Case Study Research, 44.

24
One can conclude that the several processes found in the old or evolved capitals of
Europe and US are replicated in the newly designed capitals of the 1960s in Nigeria and
Tanzania. For that, Vale clearly shows the similarity of the Washington Mall with the
planning of Abuja in Nigeria.44 Should one be cautious about falling into the Eurocentric
or Americentric approach in this theory?

Conclusion
The modernization efforts of the post-World War Two era were built upon the
search for the modern of the interwar vanguard in both architecture and city planning.
The experts of the post-colonial world together with the exponents of CIAM (Congrs
internationaux d'architecture moderne / International Congresses of Modern Architecture)
laid the ground for new visions of the city. For the first time the architect, the planner,
and the engineer were to be actively involved in the process of changing the society and
in doing so to reinvent forms of habitation, collective conviviality, and the place of the
individual within new spatial arrangements. These Decentrists, as Jacobs calls them,
were set to destroy the traditional urban life, along with the traditional house and
traditional social relations.
The modernist discourse, together with its operationalized theories, seems to have
been questioned from below. In The Modernist City Holston sets about to do just that. His
anthropological critique of the utopic city in terms of its housing arrangements and the
everydayness of communal living was intended to show the discontinuities of the
44

Vale, Architecture, Power, and National Identity, 137.

25
political project, the planning, the architecture visions, and the needs and wants of its
inhabitants. The project of social equality was considered to have failed. Vales multiple
case study also attempts to show the disconnect between the vision and its outcome.
Constructing national identity through the built environment is only an expression of the
will and political agenda of a few powerful individuals.

26

Bibliography
Berman, Marshall. All That Is Solid Melts Into Air: The Experience of Modernity. Verso,
1983.
Ceertz, Clifford. Thick Description: Toward in Interpretive Theory of Culture. The
Interpretation of Cultures, 1973, 330.
Clifford, James, and George E. Marcus. Writing Culture: The Poetics and Politics of
Ethnography. University of California Press, 1986.
Groat, Linda N., and David Wang. Architectural Research Methods. John Wiley & Sons,
2013.
Holston, James. The Modernist City: An Anthropological Critique of Brasilia. University
of Chicago Press, 1989.
Jacobs, Jane. The Death and Life of Great American Cities. Random House Digital, Inc.,
1961.
Jordanova, Ludmilla. History in Practice. Bloomsbury USA Academic, 2013.
Marcus, George E. Ethnography Through Thick and Thin. Princeton University Press,
1998.
Rabinow, Paul. French Modern: Norms and Forms of the Social Environment. University
of Chicago Press, 1995.
Vale, Lawrence J. Architecture, Power, and National Identity. Yale University Press,
1992.
Yin, Robert K. Case Study Research: Design and Methods. SAGE, 2009.
Zarecor, Kimberly Elman. Manufacturing a Socialist Modernity: Housing in
Czechoslovakia, 1945-1960. University of Pittsburgh Pre, 2011.
. The Local History of an International Type: The Structural Panel Building in
Czechoslovakia. Home Cultures 7, no. 2 (July 1, 2010): 21735.
doi:10.2752/175174210X12663437526250.

Dissertation Prospectus

Chinan 1

Altering the Socialist Space: Housing Policy Transition, Tenure, and the
Architecture of Public Housing Projects in Bucharest

The word property has no passport to cross the frontiers of the collectivist
state.1 This certainly seemed true to those who, in the early days of the Soviet Union,
took the claims of the radicals seriously, leading the way towards a classless Communist
society. During its early development, the post-revolutionary Soviet state challenged
established bourgeois property relations by nationalizing all means of production and
totally eliminating private property, including that involving land and residential
dwellings.2 At the same time, it embarked on an historical enterprise with respect to
housing stock by building and allocating state housing units to the vast majority of Soviet
citizens. After the Second World War, Romania and other countries in Eastern Europe
followed the Soviet Unions lead in these matters.
The modernist project of public housing provision in the Communist countries
still remains one of the largest state-led endeavors in human history, as it reached a large
part of the population in the Soviet Union and its satellites. Understanding the socialist
modernist approach to housing provisions is not only useful on its own terms, but also
in allowing us to uncover insights into the socialist welfare state.
It is common knowledge by now that since 1989, the free market has failed to
provide adequate housing for people in need of shelter in Romania. This is also a
1

Harold Joseph Berman, "Soviet Property in Law and in Plan: With Special Reference to Municipal Land
and Housing," University of Pennsylvania Law Review, 1948, 324. See also Walton Hamilton and Irene
Till, Property, VI ENC. Coc. Sci. 528 (last sentence of the article).
2
Except for rural property, regarding which the government allowed each family to keep the property upon
which the house was built and the land around it.

Dissertation Prospectus

Chinan 2

worldwide reality: the neoliberal creed has failed to insure adequate housing for many.
The housing question is as much a reality as it was when Engels raised the issue of
housing provision in 1872.3
The Red October raised new questions about property and reconfigured ways in
which private property and privacy were problematized. A significant part of the
literature on the Soviet Union and its Eastern European satellites emphasizes how the
Communist state used various means to control its citizens through spatial
constructions4 and how the Communist government politicized the socialist space.5
Much of it also concerns how spatial dichotomies of private/public were constructed,6
and the resulting split subjectivity or social schizophrenia of Communist life.7
In recent years, new scholarship on Soviet everyday life and property relations
has revealed previously untapped archives that provide insights into a unique nexus
between the Soviet welfare state and its citizens that strengthened residents rights
regarding housing tenure.8 I am especially interested in how these developments in social
policy permitted everyday practices to inform decisions about architectural space and
design practice within the legal, political, and economic context of the socialist system.
Such policy changes gave rise to new forms of property in socialist Romania. At first,
there were successful efforts to strengthen rights related to housing tenure; then, in the
3

Friedrich Engels, The Housing Question (Foreign Languages Publishing House, 1954).
Svetlana Boym, Common Places: Mythologies of Everyday Life in Russia (Harvard University Press,
1994).
5
Neil Leach, Architecture and Revolution: Contemporary Perspectives on Central and Eastern Europe
(Psychology Press, 1999).
6
David Crowley and Susan Emily Reid, Socialist Spaces: Sites of Everyday Life in the Eastern Bloc (Berg,
2002).
7
Oleg Kharkhordin, The Collective and the Individual in Russia: A Study of Practices (University of
California Press, 1999).
8
Mark B. Smith, Property of Communists: The Urban Housing Program from Stalin to Khrushchev
(Northern Illinois University Press, 2010).
4

Dissertation Prospectus

Chinan 3

1970s, housing laws permitted the privatization9 of the Romanian state housing stock.
One of my research foci will be an exploration of the ways in which these new property
rights shaped, informed, and influenced architectural and domestic space (housing and
urban space). Initially, I intend to examine how the new regulations concerning housing
tenure informed larger questions about state/society relations, and how residents of state
housing engaged in a dialogue with the all-powerful state. How were residents able to
express their housing rights through the adaptation and alteration of their everyday
spaces?
In order to protect and strengthen the security of housing tenure in the Soviet
Union, new governmental measures were required. The introduction of personal
property10 for the first time in the 1936 Soviet Constitution was an attempt to do just that;
Stalins new constitution sought to clarify the rights of workers to own, sell, transfer, buy,
or inherit dwellings that had been obtained through income from labor. The terms of
these new regulations on housing tenure were frequently reformulated in the Soviet Civil
Code, a process that would culminate in what many term the property dilemma:11 the

On March 28, 1973, the Romanian Great National Assembly (the countrys main governmental body)
introduced the 4/1973 Housing Bill. This measure provided that the government would finance
construction of the state housing stock through state investment funds. State enterprises financed and
managed all construction processes. This was the moment when the Romanian government introduced the
notion of personal property apartments funded by citizens (taxes) together with state credits into its housing
policy and the Romanian Civil Code.
10
Some authors define this personal property as property that provides citizens with the possibility of
satisfying their various material and spiritual needs. All, or almost all, of the objects which surround us at
home (clothing, furniture, a television, radio, refrigerator, or bicycle, and much more), and for some the
house itself constitute personal property () The right to personal property is the right of a citizen, granted
under force of law, to possess, use, and dispose of property with the aim of satisfying personal needs."
They further assert that personal property is designated for the satisfaction of the personal requirements of
citizens, and it can belong to citizens regardless of their dispositive capacity and health. Oleg
Nikolaevich Sadikov, Soviet Civil Law (M.E. Sharpe, 1988).
11
H. Kimura, Personal Property in the Soviet Union, with Particular Emphasis on the Khrushchev Era:
An Ideological, Political, and Economic Dilemma (II), Slavic Studies 14 (1970): 63119.

Dissertation Prospectus

Chinan 4

commodification of housing stock, whether state or privately owned, which led


ultimately to the rise of speculation within the emerging socialist housing market.
What was at first puzzling to me was the manner in which this personal property
terminology and policy was introduced in Romania. It turns out that the model for this
program was a 1961 ordinance in which the Soviet Union extended to all its satellites the
personal property provisions of its 1936 constitution.12 Nevertheless, it took nearly a
decade for the Romanian government to institute housing policy reform based on the
notion of personal property.
This moment represents the beginning of the existence of commercial housing
stock, and the advent of the socialist housing market. In the case of Romania, I have
discovered that property relations during the socialist years were decidedly nuanced
much more so than has been acknowledged to date in the literature on this subject13.
During the late phase of socialism, Romanian citizens privately owned most existing
personal property apartments, as was termed the type of state housing stock constructed
from the early seventies onward.
Several other issues were emerging at this time: residents were altering the space
of their apartments by demolishing interior walls, adding or removing storage space, and
opening kitchens onto hallways and living rooms. How was the work of the architect as a
designer informed by these practices? And ultimately, was the architect a bystander, a
12

Dieter Pfaff, Das sozialistische Eigentum in der Sowjetunion (Verlag Wissenschaft und Politik, 1965).
Hiroshi Kimura, Personal Property in the Soviet Union; Mark B. Smith, Property of Communists: The
Urban Housing Program from Stalin to Khrushchev (Northern Illinois University Press, 2010); P. Betts,
Private Property and Public Culture: A Forgotten Chapter of East European Communist Life, Histoire et
Politique, no. 1 (2009): 2-2; P. Charles Hachten, Property Relations and the Economic Organization of
Soviet Russia: 1941-1948 (University of Chicago, Dept. of History, 2005); Gordon L. Clark, Rights,
Property, and Community, Economic Geography 58, no. 2 (April 1, 1982): 120138, doi:10.2307/143792;
Mikhail Isaakovich Braginski, William Elliott Butler, and Avgust Afanasevich Rubanov, The Law on
Ownership in the USSR (Interlist, 1991).
13

Dissertation Prospectus

Chinan 5

direct participant, or a mediator between the state and the residents living in the socialist
housing units?
I hypothesize here that residents, architects, and the Communist state institutions formed
relational interconnection that went beyond the totalitarian nature of the Communist
system, which did indeed attempt to integrate all voices within the socialist modernist
project of housing provision.

1)

Research questions
To what extent was the socialist cityscape an outcome of negotiations between
residents and designers (architects and planners embedded within the socialist state)?

How did residents of residents living in the state-provided blocks of flats in the final
years of socialism use and misuse their property rights, and how did these everyday
actions inform housing design?

How did the modernist approach to housing impact residents everyday lives and to
what extent did their actions represent a way to build social resilience?

To what extent was the socialist housing stock used at a tool for the Romanian
Communist governments nationalizing project?

Dissertation Prospectus

Chinan 6

2) Literature Review
My research will build upon and challenge several general assumptions that govern the
current Romanian and international body of literature with respect to the Communist state
and its architectural practices. On the one hand, the Romanian authoritarian state is
perceived as having controlled its citizens at every level of their everyday lives. It is
believed that the state exercised a totalizing control over the population through the state
police as its sole means of maintaining its power.14 Additionally, many accounts assert
that the Communist state denied citizens privacy by confining them within spaces that
negated their personal freedoms, denied their individual rights, and assimilated every
transgression in terms of space modifications to the discourse of resistance and dissent.15
Within the mechanism of the state architects were perceived as mere technocrats with no
creative power, reduced to following the orders of an all-powerful state that had little
interest in the highly standardized mass production of state housing stock.16 Ultimately,
the socialist housing neighborhoods were considered non-architecture, an urban design
problem, and failures, nothing more than dormitories for the working class.17 This study
will seek to build a solid historical foundation on which to further ground analysis of the
14

Dennis Deletant, Ceauescu and the Securitate: Coercion and Dissent in Romania, 1965-1989 (M.E.
Sharpe, 1995).
15
Juliana Maxim, Mass Housing and Collective Experience: On the Notion of the Microraion [selfcontained district] in Romania in the 1950s and 1960s, Journal of Architecture 14, no. 1 (2009); Adrian
Neculau, Viaa cotidian n comunism (Polirom, 2004); Juliana Maxim, Developing Socialism: The
Photographic Condition of Architecture in Romania, 19581970, Visual Resources 27, no. 2 (2011): 154171.
16
Ion Mircea Enescu, Architect sub comunism (Bucuresti: Paideia, 2006); Ioan Augustin, Power, Play, and
National Identity: The Politics of Modernization in Central and East-European Architecture: The
Romanian File (Bucharest: The Romanian Cultural Foundation Publishing House, 1999); Gheorghe Leahu,
Bucureti: arhitectur i culoare (Editura Sport-Turism, 1989).
17
P. Lizon, East Central Europe: The Unhappy Heritage of Communist Mass Housing, Journal of
Architectural Education (1996): 104-114; M. Raluca, Understanding the Urban Past: The Transformation
of Bucharest in the Late Socialist Period, Testimonies of the City: Identity, Community, and Change in a
Contemporary Urban World (2007): 159; G. Pasztor and L. Peter, The Urban Housing Problem in
Romania: The Legacy of Communist Blocks of Flats (2009).

Dissertation Prospectus

Chinan 7

extensive socialist modernist practices, and of the role of the architect as an active force
in constructing socialist modernity.
Private/Public Space Binary in the Socialist World
In the early days of Soviet revolutionary fervor, the nationalization of property was
accompanied by the reconfiguration of state housing stock, spatial reconfigurations, and
changes in the conceptualization of private and public space: On the one hand, public and
private were collapsed by removing traditionally defined demarcations (the advent of the
communal apartment), while on the other, linguistic reconstructions of public and private
referred to etymological roots in the Russian language that were meant to distinguish
them from supposedly bourgeois Western terminology. This terminology needs to be
discussed in terms of everyday life and its connection with private versus public space.
Debates over private versus public space started with Soviet constructivist
revolutionary efforts to change understandings of private/public space in the late
twenties, as discussed by Victor Buchli in his book Archeology of Socialism (2000). In
his study of the Narkomfin Communal Apartment in the Soviet Union (Buchli 1998), he
notes that everyday life was known as byt, which is not directly translatable into
English. Loosely translated, byt encompass[es] all the following English terms: daily
life, domesticity, lifestyle, or way of life (2000, 23). And more importantly, Buchli
asserts, in the Soviet Union after the First World War, the domestic front became of
utmost importance for the socialist restructuring of society, and byt became the core of
the development of workers political and social consciousness (2000, 23). The notions of
privacy, self, and identity had no equivalent in the early Russian language, as
Svetlana Boym (1994) has shown. Boym points out that nineteenth-century thinkers

Dissertation Prospectus

Chinan 8

stressed the untranslatability of some Russian words into Western languages; she gives
the example of sobornost, which means spiritual community, indicating the Russian
mythical alternative to private life (1994). On the same note, Oleg Kharkhordin points
out that the meaning of statements that use the phrase chastnaia zhizn (a rough
equivalent of private life in English) in Russian may be radically different from those
in English or French (1999).
Prior to the 1917 Communist Revolution, individual actions were held to take
place in the private sphere, which was the authentic space, while the fake world was the
world of public complicity (Crowley and Reid 2002, 34), so it is important to question
how cultural leadership and consensus regarding socialist ideology were established,
secured and reproduced, but also negotiated and questioned at the level of everyday
practices (Luthar and Punik 2010). Luthar and Punik explain that an analysis of
power in socialism/communism should move away from its legitimate forms in central
locations to the working of power at the margins (2010). In Russia, those places
previously at the margins, in particular the domestic front, became of utmost
importance for the socialist restructuring of society.
The fall of the Berlin Wall was glorified as an important historical moment in the
development of Eastern Europe, and as a moment of victory for the West against the
Soviet Union and its satellite regimes. After this event, literature (Western and Eastern)
became forcefully critical (even more so than before the fall of the Communist system).
Many academic disciplines have discussed the failures of the socialist establishment,
together with the failure of every cultural, ideological, social, economic, and political
product that might come out of this system (Given 2008). The research presented here is,

Dissertation Prospectus

Chinan 9

on the one hand, a reaction to this generalized tendency in the current literature and, on
the other, an outcome of personal experience, as I myself am a product of the socialist
system who lived for twenty-five years in a socialist block of flats and experienced the
balcony (discussed below) first-hand. More than anything, I want to give voice to a part
of the society that has often been silenced: those people who embraced the socialist
transformations, whether tacitly or openly, and who benefited from them. The provision
of housing and improvements in quality of life (sanitation, public services, public
transportation, etc.) are just a few of the positive outcomes worthy of mention.
In reading through extant Romanian literature critical of the socialist blocks of
flats, I was amazed at the extent of exaggerated, biased, offensive (particular toward rural
migrants, the first generation of residents to live in these blocks of flats), and contrived
critiques on the part of contemporary Romanian authors regarding everyday life under
socialism, dismissing the blocks of flats as a failed, imposed, and ultimately limited type
of residential building. Fortunately, several young authors ( Petrovici and Vanea 2011;
Poenaru 2008; Bodnr 2001) have tried to recover the previously undisclosed positive
side of the socialist period in Romania, and give voice to the voiceless.
Here I will apply the private/public dichotomy to space and its socialist
interpretation, based on the old conception of socialist space as a political space. This
discussion is connected with an underlying process of change in property relations and
the introduction of the socialist concept of personal property, applied in the Soviet Union
as early as the 1936 Soviet Constitution, the adoption of which led to the institution of
homeownership in the early 1970s within Romanian housing policy. I argue that changes

Dissertation Prospectus

Chinan 10

in the ways in which the balcony was used after 1970 are closely related to the change in
tenure and a new sense of ownership promoted by the notion of personal property.
Discussions of the first generation residents living in the former socialist quarters
and specifically the ways, reasons, and meanings of their decisions to redefine the flats
spaces and their balconies in the Romanian context, is long overdue, and represents an
absence most visible in the field of architecture. During my field research in Romania in
the summer of 2011, I was not able to locate a single published work on this topic, just a
couple of unpublished masters theses by sociology students with little access to archival
sources. The most important work I have found, which I refer to quite often and which, in
part, inspired me to pursue the subject of the balcony, is Florin Poenarus book On the
Socialist Balcony: State, Citizens, and Ideology in a Romanian Neighborhood (2008a).
Written from a cultural anthropologists point of view, Poenaru presents several aspects
of the balcony enclosure phenomenon. Notably, he considers residents pragmatic
reasons and the needs behind their decisions, the ways in which they transformed and
controlled the space, and the role of the state, together with the local administration, as a
regulator. He argues that balcony enclosures were not acts of political resistance or
counter-action against the state but rather an act of adaptation to the objective
conditions of the environment, a kind of coping strategy people were forced to adopt to
make their lives bearable (Poenaru 2008, 28). Poenarus hypothesis is useful as it
provides a general framework, a departure point for an analysis of balcony enclosure. He
brings up the issues of official redistribution systems and the shortage economy as
indirect reasons for residents competing to secure food, space, thermal comfort, storage,
or living space.

Dissertation Prospectus

Chinan 11

Poenaru rightly argues that to describe socialism as an intrusion of the state in the
private lives of residents is an overstatement; he maintains that the state was not
required to be so actively involved in monitoring people (2008a, 33). Indirectly, it was
responsible for a lack of goods, material, food, and housing. He concludes that the
State/citizens relationship during socialism was characterized by a mixture of fear,
negotiations, small boycotts, adaptation, accommodations, compliance, complicity,
ambiguity, arbitrary double talk, deals, duplicity, and above all necessity (2008a, 34).
Appropriating and reconfiguring the space of the state-provided flats was an act of
everyday domestic life. It was an undertaking that represented the privacy of residents
exteriorized into the public realm through their taking over an environment that was
considered, and rightly so, a semi-public space. This ambivalence, this overlap of private
and public, was at the core of the socialist/Communist ideology that went back to the
early Soviet Union, and I specifically relate it to issues of property stemming from the
idea in Soviet Marxism that everything should be public, common good, collectively
possessed. Private property, as the name private indicates, delineates the boundaries of
privacy. As the legitimacy of private property was rejected by Marxist-Leninism, a new
concept of privacy or a private sphere needed to be introduced. Defining the boundaries
between private and public was the goal of the new regime of personal property,
introduced for the first time in the 1936 Soviet Constitution.
3) Methodology
Bucharest will be the site of my research. More specifically, I will focus on three of the
main neighborhoods: the Titan neighborhood, which was planned for a population of
200,000 residents and connected with the industrial zones 23 August-Republica and

Dissertation Prospectus

Chinan 12

Dobresti Fundeni and Dudesti-Drumul Taberei Floresti, a neighborhood with a


population of 160,000 residents, all built after 1970.18 I selected these neighborhoods
based on the chief architects involved, and because the new laws on personal property
and housing purchase through credit, introduced in 1968, applied to these new housing
units. Moreover, there is an intriguing difference between the residents of the three
locations. While Titan and Drumul Taberei are known as home to a majority of rural
migrants servicing the large industrial firms located nearby, Dobresti Fundeni had a large
number of residents from the professional and socialist elite. These three projects were
also part of the Stages of Residential Construction from 1971 to 1975 program, which
was run by head architect Ioan Ciobotaru and Sarmiza Popa, together with architect
Tiberiu Ricci (Head Architect of Bucharest), Traian Stanescu (Secondary Head Architect
of Bucharest), and Mircea Dima (Technical Head of the Institut Proiect Bucuresti
(I.P.B.).19 I have been inquiring into the contributions of sociologists whose research on
existing residential neighborhoods may help me in connecting the everyday lives of
residents with architectural design decisions.
In flipping through what was the sole architectural magazine under Communism,
Arhitectura RPR, I encountered several references to sociological research conducted in
major Romanian cities by state institutions for design and architecture. The entire body of
this research is located inside the archives of the former I.P.B., or at the IPPCT (Institutul
18

Ioan Ciobotaru, Analiza Stiintifica a Amplasarii Constructilor De Locuinte, Problema Deosebita a


Activitatii De Sistematizare, Arhitectura, 1970; Achille Ionescu, Sistematizarea Zonei Dobresti-Fundeni,
Bucuresti, Arhitectura, 1970.
19
The attractive thing about this particular program is that it did incorporate a variety of specialists. The
team also included a variety of different professionals. Among them, the engineer Simion Pop (Director of
I.P.B.), the engineer Sefcati Abdurain, the economist Valentin Banica, the architect Mariana Celac, the
sociologist Liviu Damian, the economist Florica Dumitru, the engineer Samuel Duncan, the sociologist
Felix Istrailovici, the architect Ioana Joja, and the mathematicians Georgeta Lazar and Constantin Latea.
From Ciobotaru, Analiza Stiintifica a Amplasarii Constructilor De Locuinte, Problema Deosebita a
Activitatii De Sistematizare.

Dissertation Prospectus

Chinan 13

de Proiectare Pentru Constructii Tipizate / Institute for the Design and Standardization of
Construction). Along with ISLGS (The Institute for Research and Projects for Housing,
Planning, and Communal Administration), the IAIM (Ion Mincu Institute of
Architecture) was one of the main sources for catalogs of blueprints for prefabricated
housing in Romania. It is often emphasized that the prefabricated systems valued costefficiency and short construction times above quality of life and how socialist planning
employing this system turned out to be a failure.20 This is the voice of the specialists.
What about the residents living in such prefabricated housing units? Do they share this
belief? The answers may depend very much on whom one addresses this question to.
Here, it is necessary to pause and say something about the voices of the residents.
Who are they, the subjects of my inquiry who live in blocks of flats? For my research I
am targeting three different categories of respondents. Pioneer residents, people who
migrated from a rural area and moved into the newly built blocks of flats in the early 70s,
constitute the first group. Residents relocated from urban housing slated for demolition
form the second. Children of the pioneer generation make up the third group.
Individuals will be further differentiated between renters or homeowners. Within each
group, there will be secondary differentiations between blue and white-collar workers on
one side, and on the other, professionals, bureaucrats, and party elites.
4) Research relationships
In the case of state-provided public housing, the relationship between architects,
users, and the built environment is difficult to reconstruct. How did state-built socialist
public housing involve reconciling design decisions with residents needs? Also, we need
20

D. Vais, Exporting Hard Modernity: Construction Projects from Ceauescus Romania in the Third
World, The Journal of Architecture 17, no. 3 (2012): 433451.

Dissertation Prospectus

Chinan 14

to keep in mind that architects did not have direct access to, nor directly communicate
with, the future users of the dwelling spaces they eventually designed. In its general lines,
this situation is no different from the challenges confronting public housing assistance
anywhere else. However, one issue that makes a difference is the fact that the
authoritarian state imposed its prefabricated housing units upon residents. My study
attempts to unravel the relationship between architects decisions and residents needs
and wants through a review of sociological researchfieldwork done throughout the 60s
in public housing complexes. Here, I presume that the sociological research elucidated
and informed architects design decisions, or at the very least, was intended to do so.

5) Data collection
Oral histories will provide personal stories describing the everyday lives of
residents; likewise stories from the lives and practical experiences of architects will
reveal something of their experience. Historical documentary evidence includes written
and oral sources. Oral histories allow for a researcher to construct historiographical lines,
or case histories as they are sometimes called. In the Romania of the socialist era, access
to reliable information was minimal. The government dominated all channels of
information that a researcher might be able to use for reconstructing the reality of the
socialist domestic everyday life of Romanians in an accurate manner. Archival
documents are not entirely reliable; official documents even less so. Much research

Dissertation Prospectus

Chinan 15

undertaken during the 1970s and 1980s21 by foreign researchers was full of gaps, because
they were not able to pass the gatekeepers of the socialist bureaucracy.
All the above urges the researcher to look for the first-hand information generated
by oral histories. The written sources of documentary evidence on the lives of
participants are not enough. Oral histories give voice to the common people and relate
everyday life from their vantage point, not from the perspective of those who were
engaged in running or attempting to control other peoples lives. Bruce Berg maintains
that oral histories allow researchers to avoid the deficiencies in residual and formal
presentations in documentary records through first-hand information. In dealing with
official documents, the inherent limitations of those documents are imposed on the
researcher: the documents have been filtered through official agencies and organizations
and may only contain front-stage information.22 For this work, evidence of socialist-era
everyday practices will also be sought in the personal journals of residents, their
correspondence, and in their personal photos, as well as in information gathered
regarding their political participation, and official complaints.
I expect to elicit the subjective stances of those involved in my research through
in-depth interviews conducted as part of an oral history methodology. In the case of
residents, how did they define and understand the space of the apartment they acquired
from the government as a consequence of their work? What was the significance of
owning or renting an apartment compared to their previous dwelling conditions, and how
did they go through the process of being on the waiting list? What were their

21

Richard A. French and F. E. Hamilton, The Socialist City: Spatial Structure and Urban Policy (Books on
Demand, 1979).
22
Bruce L. Berg, Qualitative Research Methods for the Social Sciences (Pearson Education Allyn &
Bacon, 2007), 276.

Dissertation Prospectus

Chinan 16

responsibilities? What was the extent of their attachment to the newly acquired flat? How
did they handle collective living and the close proximity of their neighbors? Data analysis
of interviews involves studying the nature of intersubjectivity. In analyzing this, I will
use the dialogical approach to intersubjectivity as a means of drawing connections and
correlations between the perspectives of different people. 23 To be more specific, the
concept of intersubjectivity has been used to identify agreement on the definition of an
object. The dialogical approach method is useful as an approach to interpersonal
relationships among participants who are not necessarily considered equal in the
hierarchy. Paul Sullivan, in his study of the method of dialogical analysis, claims that
such elements as the speech genre one is using in his or her stories indicates how one
orients oneself in space and time to others plus the absent others one may be addressing
when one is speaking.24 Sullivan also claims that the dialogical approach to
intersubjectivity accounts for the fact that personal participation is a dialogue with ideas
of otherssometimes dogmatic ideas that admit no dispute or sometimes with more open
ideas.25 This interaction at the level of speech allows an evolving understanding of how
residents dealt with diversity, with negotiating communal space, noise, power relations,
and friendships.
6) Research goals
The shrinking of the welfare state in Western Europe since the 1980s has reached
alarming proportions and needs to be reconsidered. I call for bringing back the welfare

23

Alex Gillespie and Flora Cornish, Intersubjectivity: Towards a Dialogical Analysis, Journal for the
Theory of Social Behaviour 40, no. 1 (2010): 1946.
24
Paul Sullivan and John McCarthy, A Dialogical Approach to Experience-based Inquiry, Theory &
Psychology 15, no. 5 (October 1, 2005): 10.
25
Ibid., 4.

Dissertation Prospectus

Chinan 17

state in a manner that allows for public and private interests to coexist, so that individual
freedom is not only about individual rights but is also about broadening choices, and the
freedom to choose from varieties of tenure that involve public housing together with
private housing provisions.26
My research goals differ in scale, and range from understanding public housing27
provision in broad terms as a top-down approach with the Communist state as the central
mechanism for resource allocation, to discussions of housing policy that will concentrate
on understanding the important policy shift of the early 1970s and the impact of a
considerable part of the states housing stock being sold to the population. I will then turn
to understanding the design institutions involved in housing provision and the role of the
architect within those institutions. Ultimately, at the micro level, I will address the
everyday domestic spaces of individual residents living in socialist blocks of flats. More
specifically, I will look at residents efforts at transforming and altering the spaces of
their state-provided apartments, and determine whether there is any connection between
their alterations and architects design decisions.
The socialist project was not limited to Eastern Europe and the Soviet Union;
much well-regarded literature has also been written on China, Cuba, Egypt, and the
former Soviet republics of Central Asia: Tajikistan, Kazakhstan, Turkmenistan, etc. The
socialist states development strategies for housing provision and the top-down approach
to public housing in particular still need to be better understood. Furthermore, the current
26

Jim Kemeny, Divergence in European Welfare and Housing Systems, Housing, Theory & Society 28,
no. 4 (2011).
27
It is also important to differentiate between public housing and social housing. The first concept is
defined as housing that is low rent, owned, sponsored (subsidized), and administered by the state. Social
housing, on the other handa concept used predominantly in the UKmeans housing provided by the
state or non-profit organizations for people of low income or with special needs.

Dissertation Prospectus

Chinan 18

housing crisis in the US and the failure of federally backed public housing opens yet
other vistas for my proposed scholarship. The socialist system, mainly in the Soviet
Union, pioneered a different approach to the notion of homeownership and to the tenantlandlord relationship. Now, more than ever, this alternative demands reconsideration.

Dissertation Prospectus

Chinan 19

Bibliography
Augustin, Ioan. Power, Play, and National Identity: Politics of Modernization in Central
and East-European Architecture: The Romanian File. Bucharest: The Romanian
Cultural Foundation Pub. House, 1999.
Berg, Bruce L. Qualitative Research Methods for the Social Sciences. Pearson Education
Allyn & Bacon, 2007.
Berman, Harold Joseph. Soviet Property in Law and in Plan: With Special Reference to
Municipal Land and Housing. University of Pennsylvania Law Review, 1948.
Betts, P. Private Property and Public Culture: A Forgotten Chapter of East European
Communist Life. Histoire@ Politique, no. 1 (2009): 22.
Bodnr, Judit. Fin de Millnaire Budapest: Metamorphoses of Urban Life. U of
Minnesota Press, 2001.
Boym, S. Nostalgia and Its Discontents1, 2007.
Boym, Svetlana. Common Places: Mythologies of Everyday Life in Russia. Harvard
University Press, 1994.
Braginski, Mikhail Isaakovich, William Elliott Butler, and Avgust Afanasevich
Rubanov. The Law on Ownership in the USSR. Interlist, 1991.
Buchli, V. Moisei Ginzburgs Narkomfin Communal House in Moscow: Contesting the
Social and Material World. The Journal of the Society of Architectural
Historians, 1998, 16081.
Buchli, Victor. An Archaeology of Socialism. Berg, 2000.
Cesereanu, Ruxandra, and Florin-tefan Morar. Romnia nghesuit: cutii de chibrituri,
borcane, conserve; (ipostaze ale ghetozrii n comunism i postcomunism) [
Cramped Romania: Box of Matches, Storage Containers, Cans (ghettoization
moments in Communism and Postcommunism]. Limes, 2006.
Ciobotaru, Ioan. Analiza Stiintifica a Amplasarii Constructilor de Locuinte, Problema
Deosebita a Activitatii de Sistematizare. Arhitectura, 1970.
Clark, Gordon L. Rights, Property, and Community. Economic Geography 58, no. 2
(April 1, 1982): 12038. doi:10.2307/143792.
Crowley, David, and Susan Emily Reid. Socialist Spaces: Sites of Everyday Life in the
Eastern Bloc. Berg, 2002.
. Socialist Spaces: Sites of Everyday Life in the Eastern Bloc. Berg, 2002.

Dissertation Prospectus

Chinan 20

Deletant, Dennis. Ceauescu and the Securitate: Coercion and Dissent in Romania,
1965-1989. M.E. Sharpe, 1995.
Enescu, Ion Mircea. Arhitect sub comunism. Bucuresti: Paideia, 2006.
Engels, Friedrich. The Housing Question. Foreign Languages Pub. House, 1954.
French, Richard A., and F. E. Hamilton. The Socialist City: Spatial Structure and Urban
Policy. Books on Demand, 1979.
Gillespie, Alex, and Flora Cornish. Intersubjectivity: Towards a Dialogical Analysis.
Journal for the Theory of Social Behaviour 40, no. 1 (2010): 1946.
doi:10.1111/j.1468-5914.2009.00419.x.
Hachten, P. Charles. Property Relations and the Economic Organization of Soviet
Russia: 1941-1948. University of Chicago, Dept. of History, 2005.
Ionescu, Achille. Sistematizarea Zonei Dobresti-Fundeni, Bucuresti. Arhitectura, 1970.
Kemeny, Jim. Divergence in European Welfare and Housing Systems. Housing,
Theory & Society 28, no. 4 (2011).
Kharkhordin, Oleg. The Collective and the Individual in Russia: A Study of Practices.
University of California Press, 1999.
. The Collective and the Individual in Russia: A Study of Practices. University of
California Press, 1999.
Kimura, H. Personal Property in the Soviet Union, with Particular Emphasis on the
Khrushchev Era: An Ideological, Political and Economic Dilemma (II).
(Slavic Studies) 14 (1970): 63119.
Kimura, Hiroshi. Personal Property in the Soviet Union, with Particular Emphasis on the
Khrushchev Era: An Ideological, Political and Economic Dilemma. Columbia
university., 1968.
Leach, Neil. Architecture and Revolution: Contemporary Perspectives on Central and
Eastern Europe. Psychology Press, 1999.
Leahu, Gheorghe. Bucureti: arhitectur i culoare. Editura Sport-Turism, 1989.
Lizon, P. East Central Europe: The Unhappy Heritage of Communist Mass Housing.
Journal of Architectural Education, 1996, 10414.
Luthar, Breda, and Marua Punik. Remembering Utopia: The Culture of Everyday Life
in Socialist Yugoslavia. New Academia Publishing, LLC, 2010.

Dissertation Prospectus

Chinan 21

Maxim, Juliana. Developing Socialism: The Photographic Condition of Architecture in


Romania, 19581970. Visual Resources 27, no. 2 (2011): 15471.
doi:10.1080/01973762.2011.568178.
. Mass Housing and Collective Experience: On the Notion of Microraion in
Romania in the 1950s and 1960s. Journal of Architecture 14, no. 1 (2009).
Neculau, Adrian. Viaa cotidian n comunism. Polirom, 2004.
Pasztor, G., and L. Peter. Urban Housing Problem in Romania: The Legacy of
Communist Block of Flats, 2009.
Petrovici, N., and A. Vanea. Consuming the City: Centrality and Class in Cluj., 2011.
Pfaff, Dieter. Das sozialistische Eigentum in der Sowjetunion. Verlag Wissenschaft und
Politik, 1965.
Poenaru, Florin. On the Socialist Balcony: State, Citizens and Ideology in a Romanian
Neighborhood. VDM Verlag Dr. Mller, 2008.
Raluca, M. Understanding the Urban Past: The Transformation of Bucharest in the Late
Socialist Period. Testimonies of the City: Identity, Community and Change in a
Contemporary Urban World, 2007, 159.
Sadikov, Oleg Nikolaevich. Soviet Civil Law. M.E. Sharpe, 1988.
Smith, Mark B. Property of Communists: The Urban Housing Program from Stalin to
Khrushchev. Northern Illinois University Press, 2010.
. Property of Communists: The Urban Housing Program from Stalin to
Khrushchev. Northern Illinois University Press, 2010.
Sullivan, Paul, and John McCarthy. A Dialogical Approach to Experience-Based
Inquiry. Theory & Psychology 15, no. 5 (October 1, 2005): 62138.
doi:10.1177/0959354305057266.
Tismneanu, Vladimir. Stalinism for All Seasons: A Political History of Romanian
Communism. University of California Press, 2003.
Vais, D. Exporting Hard Modernity: Construction Projects from Ceauescus Romania
in the Third World. The Journal of Architecture 17, no. 3 (2012): 43351.
Verdery, Katherine. National Ideology under Socialism: Identity and Cultural Politics in
Ceauescus Romania. University of California Press, 1995.

UC Berkeley College of Environmental Design Architecture

Qualifying Exam List of Readings

Chinan

Tuesday, April 15, 2014

Major: Environmental Design and Urbanism in Developing Countries


Exam Committee Members: Paul Groth (Exam Chair), Nezar AlSayyad, Greg Castillo,
Urban Studies and Housing: Ananya Roy
Art History: Heba Mostafa
1. 20TH CENTURY ARCHITECTURE AND URBANISM US AND EUROPE 2
1.1. Histories and Theories of Architecture ............................................................. 2
1.2. Critical Engagement with the Practice of Architecture .................................... 2
1.3. Social and Cultural Analysis of Dwellings ....................................................... 3
1.4. Theories of Urban Modernity ........................................................................... 3
1.5. Colonial and Postcolonial Urban Studies ......................................................... 3
1.6. Conceptions of Nationalism and the Built Environment .................................. 3
2. URBAN STUDIES AND HOUSING: THEORIES AND METHODS ............. 4
2.1. Spaces of Neoliberalism: Theories of Governance ........................................... 4
2.2. The Just City and Urban Social Movements..................................................... 4
2.3. Welfare Regimes and the Question of Tenure .................................................. 5
2.4. Urban Informality and Housing ........................................................................ 5
2.5. Public Housing Policy and Architecture ........................................................... 6
3. SOCIALIST AND POSTSOCIALIST POLITICS AND THE BUILT
ENVIRONMENT IN EASTERN EUROPE AND RUSSIA ..................................... 6
3.1. Conceptions of Socialism and Postsocialism.................................................... 6
3.2. Housing Theories and Practices during Socialism in the Soviet Block ............ 6
3.3. Everyday Architecture and the Cold War ......................................................... 6

UC Berkeley College of Environmental Design Architecture

Chinan

1. 20TH CENTURY ARCHITECTURE AND URBANISM US AND EUROPE


1.1. Histories and Theories of Architecture
1) Banham, Reyner. Theory and Design in the First Machine Age. MIT Press, 1980.
2) Collins, Peter. Changing Ideals in Modern Architecture, 1750-1950. McGillQueens Press - MQUP, 1998.
3) Frampton, Kenneth. Modern Architecture: A Critical History. Thames & Hudson,
Limited, 2007.
4) Giedion, Sigfried. Space, Time and Architecture: The Growth of a New Tradition.
Harvard University Press, 1967.
5) Hitchcock, Henry Russell. The International Style. W. W. Norton & Company,
1997.
6) Jencks, Charles. The Language of Post-modern Architecture. Rizzoli, 1981.
7) Pevsner, Nikolaus. Pioneers of Modern Design: From William Morris to Walter
Gropius. Penguin Books Limited, 1991.
8) Rowe, Colin. The Mathematics of the Ideal Villa and Other Essays. MIT Press,
1982.
9) Tafuri, Manfredo. The Sphere and the Labyrinth: Avant-Gardes and Architecture
from Piranesi to the 1970s. Mit Press, 1990.
10) Venturi, Robert. Complexity and Contradiction in Architecture. The Museum of
Modern Art, 1977.
1.2. Critical Engagement with the Practice of Architecture
11) Cuff, Dana. Architecture: The Story of Practice. MIT Press, 1992.
12) Holston, James. Insurgent Urbanism: Interactive Architecture and Dialogue with
Craig Hodgetts. In Technoscientific Imaginaries: Conversations, Profiles, and
Memoirs, 461505. University of Chicago Press, 1995.
13) Kostof, Spiro. The Architect: Chapters in the History of the Profession. Oxford
University Press, 1977.
14) Rudofsky, Bernard. Architecture Without Architects: A Short Introduction to Nonpedigreed Architecture; UNM Press, 1964.
15) Stevens, Garry. The Favored Circle: The Social Foundations of Architectural
Distinction. Cambridge, Mass.; London: MIT, 2002.
16) Till, Jeremy. Architecture Depends. MIT Press, 2009.
17) Wright, Gwendolyn, Janet Parks, and Arthur Ross Architecture Gallery. The
History of History in American Schools of Architecture, 1865-1975. Temple
Hoyne Buell Center for the Study of American Architecture and Princeton
Architectural Press, 1990.

UC Berkeley College of Environmental Design Architecture

Chinan

1.3. Social and Cultural Analysis of Dwellings


18) Bourdieu, Pierre. The Field of Cultural Production: Essays on Art and Literature.
Columbia University Press, 1993.
19) Ceertz, Clifford. Thick Description: Toward in Interpretive Theory of Culture.
The Interpretation of Cultures (1973): 330.
20) Gal, Susan. A Semiotics of the Public/Private Distinction. Differences 13, no. 1
(January 1, 2002): 7795. doi:10.1215/10407391-13-1-77.
21) Groth, Paul Erling. Living Downtown: The History of Residential Hotels in the
United States. University of California Press, 1994.
22) Webster, Helena. Bourdieu for Architects. Routledge Chapman & Hall, 2011.
1.4. Theories of Urban Modernity
23) Alsayyad, Nezar. The End of Tradition? London; New York: Routledge, 2004.
24) Berman, Marshal. All That Is Solid Melts Into Air: The Experience of Modernity.
Verso, 1983.
25) Harvey, David. Paris, Capital of Modernity. Routledge, 2013.
26) Holston, James. The Modernist City: An Anthropological Critique of Brasilia.
University of Chicago Press, 1989.
1.5. Colonial and Postcolonial Urban Studies
27) Alsayyad, Nezar. Forms of Dominance on the Architecture and Urbanism of the
Colonial Enterprise. Aldershot; Brookfield, U.S.A.: Avebury, 1992.
28) Avermaete, Tom, Serhat Karakayali, and Marion Von Osten. Colonial Modern:
Aesthetics of the Past - Rebellions for the Future. Black Dog Publishing Limited,
2010. Chapter 3- Osten, Marion von. In Colonial Modern Worlds.
29) Mitchell, Timothy. Colonizing Egypt: With a New Preface. University of
California Press, 1991.
30) Rabinow, Paul. French Modern: Norms and Forms of the Social Environment.
University of Chicago Press, 1995.
31) Said, Edward W. Orientalism. Random House LLC, 1979.
32) Wright, Gwendolyn. The Politics of Design in French Colonial Urbanism.
University of Chicago Press, 1991.
1.6. Conceptions of Nationalism and the Built Environment
33) Alsayyad, Nezar. Culture Identity and Urbanism: A Hstorical Perspective from
Colonialism and Globalisation. In Colonial Modern: Aesthetics of the Past -

UC Berkeley College of Environmental Design Architecture

Chinan

Rebellions for the Future;edited by Tom Avermaete, Serhat Karakayali, and


Marion Von Osten, 7787. Black Dog Publishing Limited, 2010.
34) Anderson, Benedict. Imagined Communities: Reflections on the Origin and
Spread of Nationalism (New Edition). Verso, 2006.
35) Hobsbawm, Eric, and Terence Ranger. The Invention of Tradition. Cambridge
University Press, 2012.
36) Vale, Lawrence J. Architecture, Power, and National Identity. Yale University
Press, 1992.

2. URBAN STUDIES AND HOUSING: THEORIES AND METHODS


2.1. Spaces of Neoliberalism: Theories of Governance
37) Alsayyad, Nezar, and Ananya Roy. Medieval Modernity: On Citizenship and
Urbanism in a Global Era. Space and Polity 10, no. 1 (2006): 120.
38) Brenner, Neil, and Nicholas G. Theodore. Spaces of Neoliberalism: Urban
Restructuring in North America and Western Europe. Wiley, 2002.
39) Crouch, Dr Colin, Prof Klaus Eder, and Damian Tambini. Citizenship, Markets,
and the State. Oxford University Press, 2001.
40) Ong, Aihwa. Neoliberalism as Exception: Mutations in Citizenship and
Sovereignty. Duke University Press, 2006.
41) Peck, Jamie. Constructions of Neoliberal Reason. Oxford University Press, 2011.
42) Roy, Ananya. Paradigms Of Propertied Citizenship Transnational Techniques of
Analysis. Urban Affairs Review 38, no. 4 (March 1, 2003): 463491.
doi:10.1177/1078087402250356.
43) Somers, Margaret R. Genealogies of Citizenship: Markets, Statelessness, and the
Right to Have Rights. Cambridge University Press, 2008.
44) Wacquant, Loc. Three Steps to a Historical Anthropology of Actually Existing
Neoliberalism. Social Anthropology 20, no. 1 (2012): 6679. doi:10.1111/j.14698676.2011.00189.x.
2.2. The Just City and Urban Social Movements
45) Caldeira, Teresa Pires do Rio. City of Walls: Crime, Segregation, and Citizenship
in So Paulo. University of California Press, 2000.
46) Castells, Manuel. The City and the Grassroots: A Cross-Cultural Theory of
Urban Social Movements. University of California Press, 1983.
47) Engels, Friedrich. The Housing Question. Co-Operative Publishing Society of
Foreign Workers (June 26, 1872).
http://www.marxists.org/archive/marx/works/1872/housing-question/index.htm.
48) Harvey, David. Social Justice and the City. University of Georgia Press, 2010.
49) Holston, James. Insurgent Citizenship: Disjunctions of Democracy and Modernity
in Brazil. Princeton University Press, 2008.

UC Berkeley College of Environmental Design Architecture

Chinan

50) Marcuse, Peter, James Connolly, Johannes Novy, Ingrid Olivo, and Cuz Potter.
Searching for the Just City: Debates in Urban Theory and Practice. Routledge
Chapman & Hall, 2011.
51) Mitchell, Don. The Right to the City: Social Justice and the Fight for Public
Space. Guilford Press, 2012.
52) Mouffe, Chantal. The Return of the Political. Verso, 2005.
2.3. Welfare Regimes and the Question of Tenure
53) Blomley, Nicholas. Unsettling the City: Urban Land and the Politics of Property.
Routledge, 2003. http://www.questia.com/library/107497209/unsettling-the-cityurban-land-and-the-politics.
54) Katz, Michael B. In the Shadow of the Poorhouse: a Social History of Welfare in
America. Basic Books, 1996.
55) Kemeny, Jim. The Ideology of Home Ownership: Homeownership Societies and
the Role of Housing. Housing, Theory & Society 28, no. 1 (2011).
56) Lennartz, Christian. Power Structures and Privatization Across Integrated Rental
Markets: Exploring the Cleavage Between Typologies of Welfare Regimes and
Housing Systems. Housing, Theory and Society 28, no. 4 (2011): 342359.
doi:10.1080/14036096.2011.562626.
57) Polanyi, Karl. The Great Transformation: The Political and Economic Origins of
Our Time. Beacon Press, 1944.
2.4. Urban Informality and Housing
58) Gugler, Josef. The Urbanization of the Third World. Oxford University Press,
1988.
59) Hall, Peter Geoffrey. Cities of Tomorrow: An Intellectual History of Urban
Planning and Design in the Twentieth Century. Wiley, 2002.
60) Mathy, Kosta. Beyond Self-help Housing. Mansell, 1992.
61) Roy, Ananya. City Requiem, Calcutta: Gender and the Politics of Poverty.
Minneapolis: University of Minnesota Press, 2003.
62) Roy, Ananya, and Nezar AlSayyad. Urban Informality: Transnational
Perspectives from the Middle East, Latin America, and South Asia. Lexington
Books, 2004.
63) Soto, Hernando de. Mystery of Capital: Why Capitalism Triumphs in the West and
Fails Everywhere Else. Basic Books, 2007.
64) Turner, John F. C. Housing by People: Toward Autonomy in Building
Environments. Pantheon Books, 1977.
65) Ward, Peter M. Self-help Housing: a Critique. Mansell, 1982.
66) Yiftachel, Oren. Critical Theory and gray Space: Mobilization of the
Colonized. City 13, no. 23 (2009): 246263. doi:10.1080/13604810902982227.

UC Berkeley College of Environmental Design Architecture

Chinan

2.5. Public Housing Policy and Architecture


67) Bristol, Katharine G. The Pruitt-Igoe Myth. Journal of Architectural Education
(1991): 163171.
68) Fathy, Hassan. Architecture for the Poor: An Experiment in Rural Egypt.
University of Chicago Press, 2010.
69) Stieber, Nancy. Housing Design and Society in Amsterdam: Reconfiguring Urban
Order and Identity, 1900-1920. University of Chicago Press, 1998.
70) Vale, Lawrence. From the Puritans to the Projects: Public Housing and Public
Neighbors. Harvard University Press, 2009.
71) Wurster, Catherine Bauer. Modern Housing. Houghton Mifflin, 1934
3. SOCIALIST AND POSTSOCIALIST POLITICS AND THE BUILT
ENVIRONMENT IN EASTERN EUROPE AND RUSSIA
3.1. Conceptions of Socialism and Postsocialism
72) Aidukaite, J. Old Welfare State Theories and New Welfare Regimes in Eastern
Europe: Challenges and Implications. Communist and Post-communist Studies
42, no. 1 (2009): 2339.
73) Burawoy, Michael, and Katherine Verdery. Uncertain Transition: Ethnographies
of Change in the Postsocialist World. Rowman & Littlefield, 1999.
74) Humphrey, Caroline. The Unmaking of Soviet Life: Everyday Economies after
Socialism. Cornell University Press, 2002.
3.2. Housing Theories and Practices during Socialism in the Soviet Block
75) Harris, Steven E. Communism on Tomorrow Street: Mass Housing and Everyday
Life after Stalin. Woodrow Wilson Center Press / Johns Hopkins University Press,
2013.
76) Smith, Mark B. Property of Communists: The Urban Housing Program from
Stalin to Khrushchev. Northern Illinois Univ, 2010.
77) Zarecor, Kimberly Elman. Manufacturing a Socialist Modernity: Housing in
Czechoslovakia, 1945-1960. University of Pittsburgh Pre, 2011.
3.3. Everyday Architecture and the Cold War
78) Betts, Paul. Within Walls: Private Life in the German Democratic Republic.
Oxford University Press, 2010.
79) Castillo, Greg. Cold War on the Home Front: The Soft Power of Midcentury
Design. U of Minnesota Press, 2010.
80) Crowley, David, and Susan Emily Reid. Socialist Spaces: Sites of Everyday Life
in the Eastern Bloc. Berg, 2002.

UC Berkeley College of Environmental Design Architecture

Chinan

81) Hixson, Walter L. Parting the Curtain: Propaganda, Culture, and the Cold War,
1945-1961. Palgrave Macmillan, 1997.
82) James, Kathleen. Bauhaus Culture: From Weimar to the Cold War. U of
Minnesota Press, 2006.
83) Pence, Katherine, and Paul Betts. Socialist Modern: East German Everyday
Culture and Politics. University of Michigan Press, 2008.

Das könnte Ihnen auch gefallen